Этого треда уже нет.
Это копия, сохраненная 6 августа 2020 года.

Скачать тред: только с превью, с превью и прикрепленными файлами.
Второй вариант может долго скачиваться. Файлы будут только в живых или недавно утонувших тредах. Подробнее

Если вам полезен архив М.Двача, пожертвуйте на оплату сервера.
Тред тупых вопросов №124 Lagrange edition 552819 В конец треда | Веб
Тред вопросов о жизни, Вселенной и всём таком.

Спрашиваем то, за что в других местах выдают путёвку в биореактор. Здесь анонимные учёные мирового уровня критически рассмотрят любые гениальные идеи и нарисованные в Paint схемы.

Предыдущий тут: >>550169 (OP)
https://2ch.hk/spc/res/550169.html (М)

Q: Можно быстрее?
A: Можно упасть в пузырь Альбукерке, NASA уже почти надула его.

Q: Я начитался охуительных историй про уфологию, че делать, нам жопа?
A: Да, тебе жопа, можешь сгонять в зогач или куда оттуда пошлют.

Q: Что будет с человеком в вакууме без скафандра / если он упадет на черную дыру / попробует ступить на поверхность газового гиганта/солнца?
A: Он умрёт.

Q: Почему бы не привязать ракету к воздушному шару или стартовать с горы?
A: Космос - это не как высоко, а как быстро, большая часть энергии ракеты уходит на разгон вбок.
Подробнее тут https://what-if.xkcd.com/58/ (английский) https://chtoes.li/orbital-speed/ (перевод)
2 552821
Можно ли создать и доставить на поверхность Венеры ровер, который проживёт хотя бы месяц? С учетом текущих технологий естественно.
3 552822
>>2819 (OP)
Насколько стабильны орбиты "троянцев" и "греков". Я помню, что точки L4 и L5 являются стабильными, то есть при небольшом отклонении объект "возвращается" на место. Но вот судя по четвертому пику там весьма некислые отклонения от точек есть.
4 552823
>>2821
Хуй знает насчёт колёс для ровера, но есть подвижки по электронике для венерианских условий:
https://www.nasa.gov/press-release/nasa-glenn-demonstrates-electronics-for-longer-venus-surface-missions
5 552829
Отличный тред для моего вопроса №2, под перекат попал лол.
Я ващет далек от конспиролохии, но посмотрел стыковку Аполлон-11 на орбите Луны и чет не розумию:
Где газ от маневровых? На видео вродь освещенная сторона, какова хуя газа не видно, он че, так сильно рассеивается и вообще свет не отражает?
Где инерция? Какова хуя Орел словно на направляющих меняет ориентацию в пространстве, и не видно вообще инерционных сил? Ну хоть чуть-чуть. Он же небось вручную стыковался, так точно вручную выдать импульс чтобы остановить вращение невозможно. Ну пусть раз, ну пусть два получилось, но не все видео же.
А что за дрожания Орла? Дрожание Колумбии, с которой производится съемка, отсутствуют, да и неоткуда им взяться - во время стыковки им маневров не совершается. А Орел сука дрожит как на тросах.
Вощем что за хуйня, поясните мне. Это реальные съемки или прост потом отсняли документальный материал а-ля "как это было"?
https://www.youtube.com/watch?v=nkUyqe2HM-8
6 552859
>>2829
Всё из-за шакального качества видео.
Ты избалован маневровыми сполохами Союза, нужно смотреть какие именно маневровые стоят на аполлонах, какой газ там выпускают, в каком количестве. Давай ты сам поищешь, ок?
На 6:08 от инерции антенна даже двигается с правой стороны модуля.

Вот еще видео с Аполлона 9 где на 12 минуте есть немного кадров стыковки
https://www.youtube.com/watch?v=xytZh161E7o
И вот еще с девятого с 36 минуты, нигде не видно сполохов маневровых даже на земной орбите
https://www.youtube.com/watch?v=HAowocKgOOU
7 552860
>>2859

> какие именно маневровые стоят на аполлонах, какой газ там выпускают,


Да обычные гидразин + тетраоксид азота, которые на выходе дают прозрачный азот и прозрачный водяной пар.

Странные движения аппаратов и видимое "отсутствие" инерции на видео объясняются тем, что оно ускорено во много раз, это не реалтайм.
8 552861
>>2819 (OP)
Траханное позорище!
Во первых абулькъерро!
Во вторых не надуть, а вытянуть, иначе постановку отменят.
В третих он должен быть очень точен, и до Луны еще далеко.
9 552864
>>2859

>На 6:08 от инерции антенна даже двигается с правой стороны модуля.


Я имел в виду движение самого аппарата. Ну то есть при начале маневра вращения относительно одной из плоскостей должен же быть более-менее плавный старт. И более-менее плавная остановка. Даже если маневровыми импульсно работают - с того видео создается навязчивое ощущение что это электродвигатели Орла вращают лол. Но и потом. Допустим что ускоренное воспроизведение объясняет такие резкие движения, но тогда ИРЛ движения были плавными и медленными, и схуяли тогда антенне двигаться. Антенна - это наверное другое, автоматическая ориентация там или еще чего на источник сигнала.

>И вот еще с девятого с 36 минуты


Где на 39 минуте и 40 секунде странные дерганья аппарата. Пожалуй, то что видео ускорено - может объяснить мои вопросы, ну канеш за исключением газа с маневровых. Вот этот >>2860 анон вроде бы прав насчет используемого маневрового топлива вижу какие-то знакомые слова, лол. Но водяной пар должен кристаллизоваться ведь и хоть как-то отражать свет. А хуй. Ууъуъу сьука.
10 552866
>>2829
Выхлоп от маневровых ДУ виден далеко не у всех кораблей и далеко не при всех маневрах. И для этого нужен в основном почти контровый свет на фоне неба. Кроме того, если правильно помню, при удержании ориентации (режим ATTH, типа killrot в орбитере или кнопки T в KSP) аполлоновский ECA еще и переключался на импульсно-модулированный режим для клапанов RCS при достижении мертвой зоны угловой скорости, поэтому пшики в финальных этапах сближения всегда мелкие. Могу покопать документацию, но вроде так.
11 552867
>>2866

>Могу покопать документацию


Нет, не надо. Мы же не в зогаче в конце концов, а ответы на свои вопросы я более-менее получил, как минимум базу для построения логичного объяснения.
Спасибо вам, спейсаны.
12 552868
>>2822
Стабильны-то они стабильны, но это не значит что они тянут к себе что-либо. Эти говна описывают довольно широкие восьмерки и прочие квазиорбитальные фигуры вокруг некоей мнимой точки в системе координат Земли.
13 552872
>>2864

>Пожалуй, то что видео ускорено - может объяснить мои вопросы



На 39:34 следи за движениями облаков позади аппарата как они сначала медленно идут, а потом вдруг ускоряются вместе с ускорениями движения аппарата, явно ускоряли.
https://www.youtube.com/watch?v=HAowocKgOOU
Там же кажется вся эта ебучая плёнка, физическая плёнка, которую в катушечках держали в камере, от скорости движения рулона при записи зависел фпс наверное, а потом при оцифровке могли еще при проигрывании с разной скоростью крутить, или же какой-то оцифровщик намерянно решал сократить размер файла чтобы ускорить что-то или сэкономить место
Animationof2010TK7.gif7,7 Мб, 560x420
14 552874
>>2822
>>2868
Типикал квазиорбита в тех краях
15 552897
>>2821

>Можно ли создать и доставить на поверхность Венеры ровер, который проживёт хотя бы месяц


Можно

>С учетом текущих технологий


Нет. Современные технологии долго и упорно ломились в одну узкую нишу и вне её слабо применимы. Разработка устройств ля экстремальных условий происходит почти с нуля чаще всего.
sage 16 552898
>>2829

>Где газ


Вокруг посмотри
17 552907
>>2874
Спасибо. Понятно, что точки сами по себе ничего не тянут, там же массы нет, просто за счет комбинации гравитации двух тел создается некое движение в окрестности этих точек. Я просто не ожидал, что окрестность такая большая.
18 552925
>>2874
А где можно как на пике с телами играться и залипать на их траектории как там вот?
19 552928
>>2925
Прям "как там вот" - в GMAT
https://sourceforge.net/projects/gmat/
Пиздатейший симулятор, рекомендую. Но для космических аппаратов. Для астрономических объектов есть Universe Sandbox, но вообще думаю что конкретно эта гифка была нарисована в питоне наверняка, при помощи одного из астродинамических пакетов, или вообще по формуле.
20 552981
>>2925
https://glitch.com/~pool-sleep-5uryon6abm
У меня вот такая есть
21 553026
>>2819 (OP)
https://www.nature.com/articles/s41550-019-0906-9

Таки замкнутая значит? Каков размер тогда?
22 553028
>>2819 (OP)
Ебанный стыд...
Во-первых, Алькубьерре.
Во-вторых, не упасть, а создавать вокруг корабля изнутри (иначе кина не будет).
В-третьих, НАСА искривляет пространство на десятимиллионную часть, контролируя это сверхточными интерферометрами, до самого варп-привода здесь - как до Антарктиды раком.
23 553031
>>3026

Так чё? Вселенная таки замкнутая? Да?
24 553046
>>3026
>>3031
Да пиздят поди
25 553061
>>2874
Объясните почему переодически колбасит землю? Кривизна орбиты? Но тогда почему это происходит не всё время, а с перерывами? циклы миланковича же десятки тысяч лет а не столетие полтора.
26 553075
Если 99.9999% топлива ракеты расходуется на набор орбитальной скорости, то зачем нужны запуски ракет с самолётов и проекты по типу катапульт и пушек? Для одноступенчатых ракет? Для запуска без стартового стола? Потому что с 20-30 км легче стартовать?
27 553076
>>3061
Да

>>3075
Нет
28 553077
>>3061

>Объясните почему переодически колбасит землю?


В смысле "колбасит"?

>>3075

>зачем нужны запуски ракет с самолётов и проекты по типу катапульт и пушек?


Катапульт и пушек НЕ ТУ ТВЕРДО И ЧЕТКО. На луне может быть и возможно запилить такой старт, на земляшке - нет.
С самолетов т.к. можно на любое наклонение и на большом расстоянии от космической инфраструктуры стартовать. Под окно гадать сильно не надо, чтоб космодром четко в нужном месте был, хочешь сегодня пулять - можно 200км туда-то отлететь и стартовать. На следующей неделе? 200км в другую сторону и пуляй.
Основной профит в вариантах направления старта.
С мыса Карнавала пулять можно только на восток, а с Вварденфелла на сервер, грубо говоря, а с самолета OTLETEL PODALSHE и пульнул нахуй.
29 553079
>>3046
По мне так думается, что это самый лучший вариант.

Чёрный шарик снаружи определённого размера, который может и расти и сжиматься, а внутри как в игре при выходе за предельные значения некого пространства элемент переносится, а по факту просто отображается в другом месте.
30 553082
>>3077

>С мыса Карнавала пулять можно только на восток, а с Вварденфелла на сервер, грубо говоря, а с самолета OTLETEL PODALSHE


Если ты действительно борешься за эффективность, то на полярные орбиты будешь запускать с самолёта, летящего на широте Варденфелла, а на восток с экватора. Неизбежно.
31 553098
>>3077

>В смысле "колбасит"?


На изображении синяя точка дёргается из стороны в сторону.
Можно предположить, что это кривизна орбиты синей точки. Однако она не всегда двигается туда-сюда, а с перерывами.
К чему бы это?
32 553101
>>3061
>>3098
Ты посмотри на масштаб лет на гифке. У Земли орбита не круговая и возмущенная, трястись будет в любом случае.
33 553102
>>3075

>зачем нужны запуски ракет с самолётов


Ради мобильности точки пуска, ради запуска без стартового стола. И да, поскольку масса ракеты растёт экспоненциально от дельты, приличная доля массы экономится таки.
34 553103
>>3101
Я слышал только о циклах миланковича, которые происходят с периодом десятки и сотни тысяч лет, а не просто сотни. Что это за возмущения такие?
35 553106
>>3103
Какие еще циклы миланковича, причем тут они? Циклы Миланковича относятся к инсоляции, они никак не влияют на орбиту. Они вызваны её характеристиками - периодичностью параметров, гравитационными возмущениями и т.п.

>Что это за возмущения такие?


Гравитационные возмущения орбиты, вызванные притяжением других тел, особенно Юпитера. Это только в учебниках тела летают по эллипсам, на практике тел дохуя и они тянут друг друга в разные стороны. Строго говоря, никто не знает даже, стабильна ли орбита Земли, да и Солнечная система вообще, дальше нескольких десятков миллионов лет вперед.
36 553110
>>3106
Вот не надо этого циклы миланковича это изменение экстриситета под действием в основном юпитера, в следствии которого происходит изменение инсоляции, а не просто изменение инсоляции с нихуя.
Что-то мне не верится, что есть какой-то периодический процесс раз в 100 лет, относящийся к земле, и который не исследовали уже вдоль и поперёк.
image.png104 Кб, 447x246
37 553113
что мешает взять углерод и облучив его альфа и бета излучением получить кислород?
что мешает притащить на марс астеройд из углерода и превратить его в кислород?
38 553115
>>3110

>Вот не надо этого циклы миланковича это изменение экстриситета


>Milankovitch cycles describe the collective effects of changes in the Earth's movements on its climate


По определению. Почитай, что именно открыл Миланкович, кто он был и почему это назвали его именем.

>изменение экстриситета под действием в основном юпитера


Это причина этих циклов. Об этом и речь.

>Что-то мне не верится, что есть какой-то периодический процесс раз в 100 лет, относящийся к земле, и который не исследовали уже вдоль и поперёк.


С чего ты решил, что он есть? Если ты про трясущуюся Землю на гифке - это резонанс эллиптичности орбиты и периода кадров, возможно еще каких-то процессов, учтенных конкретно в той симуляции. Но главным образом биения дискретизации кадров гифки.
39 553116
>>3113
Мешает требуемая энергия, количество полученного, радиоактивность полученного изотопа
Если ты сможешь CN-цикл поддерживать, у тебя никаких проблем не будет с доставкой кислородия
image.png20 Кб, 782x759
40 553123
что мешает взять и привезти на орбиту земли по частям огроменнейший космический танкер, накачать его до отвала воздухом земли, опустив с орбиты шланг, а потом с помощью луны и троса запустить себя на марс с ++++ скоростью так, чтобы танкер разбился и освободил кислород? или сделал гравитационный маневр, сбросив скорость хз. принцип ускорения на картинке как объяснить хз.
41 553124
>>3123
Да ничего не мешает, просто один такой это будет пук в никуда.
Для масштабов целого Марса - мешает гигантский объем перевозимого и ограниченность земной атмосферы. Ну и по мелочи, отсутствие цикла, восполняющего/сохраняющего кислород.
42 553125
>>3124

>>гигантский объем перевозимого


да большой, но зато есть невесомость. можно сделать большой пакет, который раздуется к примеру. можно сжать. можно просто строить ооочень большой корабль. ты не ограничен.

>>ограниченность земной атмосферы


марс меньше земли. можно по братски поделить напополам атмосферу. может хватит для жизни растений, а это уже пол дела

>>отсутствие цикла, восполняющего/сохраняющего кислород.


можно все засадить там растениями, животными и вот тебе цикл.
43 553126
>>3123
1.Трос надо делать из неведомого вещества выдерживающий собственный вес.
2.Сам трос имеет массу, которую надо запустить в космос. И что-то мне кажется что лучше уж сразу кислород запускать.
3.Трос имеет площадь поверхности большей чем площадь поверхности самого корабля. Соответственно трос затормозится, начнёт падать на землю с кораблём или без . И скорее всего сгорит от трения.
4. трос длинной 380 000 км не выдержит даже собственный вес какими бы футуристичными материалы не были. 35 000 уже считается фантастикой. Трос 380 000 из нанотрубок будет толщиной с видимую часть вселенной.
5 Трос не зацепится за луну, ну или надо будет прокопать всю луну до мантии и сделать из него супер крюк что не отвалится под силой этого танкера. Опять же легче на ракетах.
6.Скорость вращения луны недостаточна дабы добратся до марса ракеты всё равно понадобятся для доускорения и по прилёту на торможение.
7.Ну долетишь ты до марса. Танкер разобьётся и выпустит одну квинтилионную долю атмосферного воздуха, который даже не кислород, а азото-кислородная смесь.
8.Ну привёз ты квинтилион танкеров с атмосферой земли. Углекислый газ атмосферы марса всё так же будет тебя удушать и дышать ты там не сможешь.
9. И вот ты привёз секстиллионы танкеров на марс. Хорошо на марсе жить можно. А на земле уже нет.
>>3125
Атмосферное давление слышал, не? по братски поделишься и на земле будут только мхи да лишайники высокогорья. Атмосфера как бы не увеличивается от растений, они просто берут CO2 и забирают C, а потом это C превращается обратно в CO2 под действием разлагающих бактерий и грибов. объём воздуха не увеличивается.
44 553127
>>3098
У земли афелий 152 гигаметра, перигелий 147 гигаметра.
Каждый год земля на 5 миллионов километров то ближе, то дальше к солнцу.
45 553128
>>3126
ты меня наверное не понял, но и корабль, и трос будут в космосе, а значит никакого трения, веса и тд не будет. Зацепиться можно будет обогнув луну в лассо как у ковбоев)) да и не одну квинтилионную можно перевезти. объемы корабля не ограничены особо + можно сжимать воздух.
а на счет выработки кислорода. я только что читал, что фотосинтез это свет + вода = кислород. растениям нужна только вода и будет пополнение атмосферы в полноценную и неясно чем тебе не нравится азото-кислородная смесь? как будто ты чем то другим сейчас дышишь. ну а на землю пофиг (упадет на 20-30% процентов да на эверест не залезть, пару миллионов слабачков помрут, но вскоре она должна восстановится потому что я писал выше.
46 553129
>>3123

>что мешает взять и привезти на орбиту земли по частям огроменнейший космический танкер,


Цена, отсутствие инфраструктуры позволяющей это делать в разумные сроки.

>накачать его до отвала воздухом земли, опустив с орбиты шланг


1. Шланг большой длины будет весить как сам танкер, даже если из нанотрубок
2. На орбитальной скорости его порвет + он сгорит в атмосфере
3. Он представляет собой охуенный тормоз, сведя танкер с орбиты в кратчайшие сроки
4. Нельзя качать насосом находящимся сверху выше определенного предела: нельзя качать атмосферу из космоса, нельзя качать воду из трубы высотой выше 10 метров, нельзя качать ртуть из трубки высотой выше 780 миллиметров.
И многое другое.

>, а потом с помощью луны и троса запустить себя на марс с ++++ скоростью так, чтобы танкер разбился и освободил кислород?


Прочность троса, опять же, достаточно слингшота, но танкер в любом случае должен быть управляемый, иначе промахнется без коррекций на полпути.
Но самое главное - попадание йобы в марс на скорости выше скорости убегания просто выкинет обломки и кучу газа с поверхности нахуй. Плюс, кислород окислит все что только можно.
Плюс для создания атмосферы танкер должен быть размером с планетоид и единовременное столкновение с такой массой будет катаклизмом.
47 553130
>>3126
Кек, ты так же ответил.

>>3128

>корабль, и трос будут в космосе, а значит никакого трения, веса и тд не будет


Ты говорил шланг. Который опускал в атмосферу. Который является охуенным тормозом.
Будет еще какое.
Кстати, еще момент, который мы упустили - огромный "танкер" посбивает спутники своим телом и должен быть дохуя прочным, т.к. из-за многокилометровых размеров будет испытывать заметные приливные силы.
>>3128

>а на счет выработки кислорода. я только что читал, что фотосинтез это свет + вода = кислород.


Нет, это некоторые бактерии воду расщепляют. Растения поглощают СО2, используя С для роста и выделяя кислород. Днем, во время фотосинтеза. Ночью они полученные днем сахара сжигают так же как и аэробные организмы.

>растениям нужна только вода и будет пополнение атмосферы в полноценную


А еще давление, температура, многочисленные параметры грунта и симбиоз с микроорганизмами.

>и неясно чем тебе не нравится азото-кислородная смесь?


Он о том, что ты чистого кислорода не везешь, как ты писал в оригинальном посте.

>ну а на землю пофиг (упадет на 20-30% процентов да на эверест не залезть, пару миллионов слабачков помрут, но вскоре она должна восстановится потому что я писал выше.


Не восстановится. Атмосфера ниоткуда не берется, она есть и потихоньку сдувается. Пополняться атмосфера сама может только за счет геологической активности, вулканизма, и основной газ который выходит в таких случаях - you guessed it - CO2.

Если ты собрался на Марсе атмосферу запиливать - ебашь водно-аммиачными кометами по нему. Программа эта на много тысяч лет. Венеру можешь заодно поколбасить.
image.png93 Кб, 307x166
48 553131
49 553133
>>3131
Не парься, Илон, астероидные ISRU и долговременные хабитаты тебе помогут продержаться пока марс будешь терраформировать.
50 553146
>>3110
Время Ляпунова применимо к Земле так же как и к другим объектам. А у Земли оно еще и точно не известно. То есть все еще хуже
43.jpg23 Кб, 450x450
51 553149
Почему на Луне такие покатые холмы?
Атмосферы нет, гравитация ниже. Из-за чего порода эррозирует?
52 553155
>>3149
Эрозия с одной "р".
И глагол "эродировать" от "erode" - https://ru.wiktionary.org/wiki/эродировать

Холмы как раз и будут рассыпчатыми и покатыми в отсутствие эрозии, анон. Каньоны и резкие уступы созданы на нашей планете благодаря эрозии, а не вопреки ей. Ещё один механизм мог бы быть - живая и очень активная тектоника, которой у луны не пахнет.
Потому весь рельеф образован тем, что отсыпется от взрыва. Поверхность луны суть - набор кратеров.
53 553161
>>3149
это горы очень колючей твёрдой пыли. мы привыкли что пыль мягкая или мелкая, но там это просто колючий толчёный камень, который миллионами лет падал сверху от ударов и выбросов породы.
54 553164
Я под впечатлением от этого видео:
https://2ch.hk/spc/src/425968/15822431953990.webm (М)
https://2ch.hk/spc/src/425968/15826701220400.mp4 (М)
Неужели всё так и будет? Что сделать-то можно?
55 553165
>>3164
Может будет, а может нет. Раньше люди думали что можно за край Земли выпасть.
56 553166
>>3164
Не будет, человечества не будет, не переживай.
57 553182
>>3164
Всегда мне казалась это видео каким-то сектанским.
58 553188
>>3164
Не ссы, ты всё равно умрёшь гораздо раньше, эти проблемы уже не будут твоими. Пусть постхьюманы себе этим мозги ебут.
59 553190
>>3164
Во-первых, наша космология даже в рамках одной галактики пока полна дыр и условностей, а в масштабах Вселенной это вообще гадание на кофейной гуще. Мы даже с расширением пока ещё окончательно не определились, есть ли оно глобально вообще, ускоренное ли оно, и сколько там на самом деле производных.

Во-вторых, тепловая смерть вселенной - процесс ассимптотический. Даже если разность потенциалов крайне мала - она никогда не равна нулю, и динамические самоорганизующиеся системы продолжат существовать. Допустим, звёзды давно выгорели, всё остыло, и вселенная заполнена ошмётками чёрных карликов температурой 0.0001К, совершающих одно малюсенькое колебание за миллиард лет. Это для нас миллиард лет. А для них это микросекунда, и никакой тепловой смерти они даже не заметили. Например, если сравнивать с кварковой эпохой, то тепловая смерть вселенной давно произошла. Может там даже зверушки какие-то жили на неизвестных физических принципах, чьи сотни поколений умещались в одну нашу микросекунду.

В-третьих, есть такой прикол, как время возврата Пуанкаре. И хотя он оперирует ОЧЕНЬ большими масштабами времени, это такое неизбежное математическое следствие существования мира. Когда-нибудь через N лет (где N всё ещё намного меньше числа Грэма) ты, анон, снова родишься и будешь писать на спейсач. А потом снова родишься. И во всех возможных вариациях.

Ну и в-четвёртых, есть старое доброе эвереттовское самоубийство, которое можно понимать расширенно. Если где-то есть наблюдатель, то совершенно похуй, где и из чего сделано это самое "где", будь оно хоть параллельным миром, хоть тем же миром через время возврата Пуанкаре, хоть пузырём на соседней бране, хоть результатом симуляции. Наблюдатель обречён существовать, поскольку вариантов, где он перестал существовать, он пронаблюдать не способен.

Короче, не рефлексируй по всякой хуите. Поважнее дела есть.
61 553208
>>3190
Космачую.
62 553209
>>3190

>Во-первых, наша космология даже в рамках одной галактики пока полна дыр и условностей


А каких именно?
63 553211
>>3149

>Из-за чего порода эррозирует?


Эродирует. Из-за постоянных ударов метеоритов образуются осколки и пыль. А поскольку у Луны все же заметно ненулевое тяготение, значительная часть падает вокруг, а не улетает в ебеня. Из-за солнечного воздействия: днем (лунным) порода нагревается, ночью остывает, что приводит к растрескиванию и измельчению.
64 553213
>>3164
Какая разница, все равно помрем раньше из-за короновируса. В живых останутся только хикке-сычи, а они не размножаются.
65 553221
>>3209
Проблема вращения галактик, из которой родился ненаблюдаемый костыль в виде тёмной материи и целая пачка альтернативных теорий гравитации типа MOND,TeVeS и.т.п. Дальше пошли всякие упоротые экстраполяции и экстраполяции экстраполяций, которые пока ни проверить, ни опровергнуть. Есть даже такие версии, что вся наблюдаемая Вселенная находится внутри охренительно большой чёрной дыры, и целые пачки далеко идущих выводов. Вплоть до космологических червоточин и N>3-мерной вселенной за пределами горизонта событий, который сплющил всё лишнее в планковский размер.

>>3200
Вот это одна из таких экстраполяций, порождённых другим костылём, которую распиарили, потому что она выглядит страшно и зловеще. Почти как распад ложного вакуума и гравитационный антивзрыв великого аттрактора. Такая-то космологическая крипипаста.
По факту у нас большие проблемы даже с оценкой скорости далёких объектов. Мы отлично можем измерить скорость "на нас" и "от нас" допплеровским способом, но у нас большие проблемы с измерением скорости вбок.

Например, мы запросто можем оказаться в квазистационарной неравномерно-пульсирующей вселенной, в которой различные области пространства (размерами в миллиарды световых лет) попеременно сжимаются и расширяются. Находясь в единственной точке и в единственной фазе мы вообще ничего не можем сказать наверняка, а телепорт в другие нам пока не завезли.
66 553230
>>3221

>ненаблюдаемый костыль


Интеллект твой ненаблюдаем. Проследуй в зогач.
67 553231
>>3221
Есть кривая скорости расширения в разные возраста вселенной. Наука не стоит на месте.
2gExnXvnpoU.jpg32 Кб, 960x704
68 553233
>>3230
Звоните в нобелевский комитет, двачер обнаружил тёмную материю у себя в очке.

>>3231
Если ты про пикрелейтед, то это всего лишь обоснование радиального движения пространства, а не радиального разлёта галактик по неподвижному пространству. Про тангенциальные скорости она вообще ничего не говорит. Да и гарантий, что постоянная Хаббла на самом деле одномоментно-одинакова во всех точках наблюдаемой вселенной, не даёт. Мы вообще не в состоянии наблюдать то, что там в данный момент происходит.
69 553236
>>3233
Я оговорился Есть кривая ускорения вселенной(хотя в общем-то одно и тоже). Очевидно мы не в центре вселенной, и потому скорость убегания конкретной области от нас так же велика как скорость убегания той точки от другой точки с такого-же расстояния. А галактики не очень-то и быстры относительно процессов крупномасштабной структуры вселенной. И скорости эти мерят по сверхскоплениям, а не по отдельным галактикам. И в общем-то зная среднюю скорость компонентов , чисто математически получаем скорость сверхскопления относительно нас, с минимальной погрешностью, которые считай не двигаются. И на их доплеровское смещение влияет расширение, а не гравитация, которая на таких масштабах ничтожна, если вообще существует.

Да и галлактики с нихуя не разлетаются хотя бы потому, что нет центра из которого надо разлетаться.
70 553246
Насколько было бы выгодней запускать небольшие космические аппараты не с земли, а со стратостатов? Скажем с 30 километровой высоты.
71 553251
>>3246
Категория: Платиновые вопросы ТТВ
Посмотри ответ в самом низу шапки треда.
72 553290
>>3164

Как-то ШУЕшно
73 553296
>>3251
Я видел, но нормального ответа там нет.
Во-первых, в вакууме жрд работают намного эффективнее, чем при атмосферном давлении.
Во-вторых, больше высота - меньше вес, больше удельная весовая тяга.
В-третьих, значительно меньше аэродинамических потерь.
В-четвёртых, можно сразу стартовать под меньшим углом к горизонту, и с бОльшим ускорением.
В итоге общая дельта, требуемая для выхода на орбиту, должна быть меньше чем при старте с поверхности.
Вопрос на сколько, примерно?
74 553297
>>3296

>Во-первых, в вакууме жрд работают намного эффективнее, чем при атмосферном давлении.


Неправда. 300 против 330 секунд обычно, т.е. 10% в лучшем случае.

>Во-вторых, больше высота - меньше вес, больше удельная весовая тяга.


Неверно. Ты разгоняешь не вес, а массу. Масса не меняется.
И даже если типа выше легче - на 400км, высота МКС, вес составляет 90% от веса на поверхности.

>В-третьих, значительно меньше аэродинамических потерь.


Верно. Это один из плюсов. Сыграет ли он против минусов такого подхода?

>В-четвёртых, можно сразу стартовать под меньшим углом к горизонту, и с бОльшим ускорением.


Второй плюс, но про ускорение хуита.

>В итоге общая дельта, требуемая для выхода на орбиту, должна быть меньше чем при старте с поверхности.


Где-то на процентов десять в самом лучшем случае.

>Вопрос на сколько, примерно?


Процентов десять.
И это в самом идеальном случае. Факторов дохуища, и все эти бонусы от высокого старта нивелируются огромной кучей минусов воздушного старта.
75 553298
Почему восход солнца ровно за моей спиной и прямо передо мной луна которая освящена лишь на половину? Она должна быть целиком сетиться а не напополам.
Как это работает?
Вот нихуя не работает и потом удивляетесь почему люди выбирают плоскую землю чем ваш пиздежь
76 553299
>>3296>>3246
Если запускать ракету со статично висящего аэростата, то экономишь только на аэродинамическом сопротивлении и небольшом начальном бусте к УИ. Стартовать в любом случае придётся вверх, ибо сразу ты горизонтальную скорость для инерционного "висения" над убегающей поверхностью не наберёшь. Топлива реально потребуется меньше, но ненамного. Процентов на 5-10. Т.е. тебе в любом случае придётся тащить в стратосферу несколько сотен тонн. Масшатабирование существующих стратостатов даёт требуемый объём баллона примерно в 0.25-0.3 кубических километра!

Короче, идея - полная хуита. Можно заменить стратостат на очень жирный самолёт - это уже будет технически воплотимо, но всё равно экономия маленькая. А вот если взять большой и охуенно быстрый самолёт, хотя бы на 5-6М, то будет уже 2..4 кратное уменьшение как требуемого топлива, так и требуемой мощности маршевого двигателя.
77 553300
>>3297

>Неправда. 300 против 330 секунд обычно, т.е. 10% в лучшем случае.



Справедливости ради, 330 против 300 это с соплом, оптимизированным под давление на уровне моря, с 30 км можно будет стартовать сразу с гораздо более широким вакуумным соплом. Для примера цифры от спейсиксовского Мерлина-Д:

С атмосферным соплом со степенью расширения 16:
- на уровне моря: 282 с,
- в пустоте: 311 с.

С вакуумным соплом со степенью расширения 165:
- в пустоте: 348 с, т.е. разница уже не 10, а 23%.
78 553301
>>3300
Соглашусь, выгода есть, и я не говорил что ее нет.
Но гемор с воздушным стартом все равно того не стоит.
79 553304
>>3298
Что надо сделать что-бы луна была освещена полностью. Надо находится +- ровно между солнцем и луной. И либо ты улетаешь в космос и наблюдаешь там полную луну, либо ждёшь когда луна окажется за землёй на которой ты находишься. Т.е. на ночной стороне. В это время луна будет полной, но солнца ты не увидишь ибо ночь.
И вот ты наблюдаешь солнце на восходе\закате. Проведи линию от солнца через себя и обнаруж, что для того что бы видить полную луну ты должен смотреть за горизонт. Либо солнце выше горизонта либо луна. Когда видна и луна и солнце, то соответственно ты не находишься между луной и солнцем.
И т.к ты скорее всего не на экваторе и солнце у тебя не встаёт ровно на востоке и не садится ровно на западе, в то время как луна летит по своей орбите с не нулевым наклонением, то скорее всего не на одной линии и луна на каком нибудь запада-запада-юге, а солнце на каком-нибудь юго-востоке.
Прежде чем опровергать круглую землю следует опровергнуть геометрию.
80 553305
>>3304
А почему луна всегда вертикально освещена? Почему нет чтобы луна под углом освещена была, всегда четко сверхувниз граница?
мавритания.png107 Кб, 320x512
81 553313
>>3305
На экваторе линия терминатора, т.е. граница света и тени проходит слева на право. И в зависимости от твоего расположения будет разный угол относительно поверхности. Например: Петербург находится на 60 широте. В момент равноденствия, в то время когда луна находится в узле своей орбиты, угол терминатора и земной поверхности будет повёрнут на 60 градусов. Соответственно летом будет уменьшатся и становится более острым, и когда луна будет в перигее и апогее, то увеличиватся то уменьшатся. В первом случае с амплитудой 23.5 градуса и лунная орбита что-то около 5 градусов.
А почему тебе кажется что луна всегда ровно светит мне неведомо. Возможно по той же причине по которой перевёрнутый вверх ногами или набок текст не осознаётся как перевёрнутый. И вообще это европоцентризм думать что сверху вниз, а не слева на право. Вот тебе флаг Мавритании с изображением луны.
82 553328
Вопрос был про вывод совсем небольших спутников на опорную орбиту. Максимум сотня килограмм полезной нагрузки.
>>3297

>про ускорение хуита


Раз не надо преодолевать сопротивление воздуха, "лишняя" прочность позволит более высокую перегрузку.

>Процентов десять


Это немало так-то.
>>3299

>Т.е. тебе в любом случае придётся тащить в стратосферу несколько сотен тонн


Не надо сотни тонн, вопрос был про маленькие аппараты.
Стартовая масса будет несколько тонн (ну пусть десять тонн).

>очень жирный самолёт


Стратостат вменяемых размеров будет, вероятно, в сотни раз дешевле такого самолёта.
>>3300

>т.е. разница уже не 10, а 23%


Уже не так уж и мало, не правда ли?
83 553330
>>3328

> Вопрос был про вывод совсем небольших спутников на опорную орбиту. Максимум сотня килограмм полезной нагрузки.


Гугли Pegasus и Launcher One. 300-400 кг, страт с 747. Или, лол, загугли zero2infinity, те ещё омичи
84 553341
>>3328

>Раз не надо преодолевать сопротивление воздуха, "лишняя" прочность позволит более высокую перегрузку.


Сопротивление воздуха имеет значение только для обтекателя, плюс перегрузка не является важным ограничивающим фактором для ракет. Ничего не мешает ракетам стартовать хоть с ускорением в 10g, просто ставить такие мощные двигатели тупо невыгодно с точки зрения максимизации полезной нагрузки, они слишком много весят. Любая вменяемая орбитальная ракета будет взлетать хоть с уровня моря, хоть с горы, хоть со стратостата с TWR в пределах 1.2~2.

>Это немало так-то.


Это практически нихуя по сравнению с дополнительной сложностью воздушного старта и невозможностью использовать более энергетические криогенные топлива.

>Стратостат вменяемых размеров


Стратостат вменяемых размеров это оксюморон, из-за ничтожной плотности воздуха и, соответственно, ничтожной подъемной силы газа на таких высотах в любом случае требуются шары огромного объема. Тот же Red Bull Stratos поднимал около тонны веса при объеме в 850 тысяч кубических метров, у тебя даже с небольшой ракетой выйдет шар под десяток миллионов кубометров.

>т.е. разница уже не 10, а 23%


Это разница в удельном импульсе только на начальном этапе работы первой ступени, уже через минуту после старта ракета в любом случае будет лететь в практически полном вакууме. Разница в дельте будет гораздо меньше. Хотя возможность использовать вакуумное сопло для первой ступени это неплохой плюс, да.
85 553352
Анон, а вот есть обсерватория DSCOVR

>наблюдает нашу Землю с расстояния 1,5 млн км, располагаясь в точке Лагранжа L1 системы Земля-Солнце. Аппарат расположен так, чтобы всегда наблюдать планету с освещенной стороны, съемка ведется с частотой примерно один кадр в два часа. В его задачу входят метеорологические наблюдения и предупреждение о солнечных вспышках.


Почему раз в два часа? Скорость передачи данных низкая для обьема картинки, что он генерирует? Что-то изнашивается на самой обсерватории? Вряд ли там механический затвор. Или просто нет необходимости чаще снимать? Интересует прежде всего, была бы разница принципиальная между двумя часами и 10 минутами например.
86 553357
>>3328

>Не надо сотни тонн, вопрос был про маленькие аппараты.


>Стартовая масса будет несколько тонн (ну пусть десять тонн).


>Стратостат вменяемых размеров будет, вероятно, в сотни раз дешевле такого самолёта.


Не будет.
https://ru.wikipedia.org/wiki/Red_Bull_Stratos
Сабж при нагрузке в 1.4т имел объём 850000м3. Это, на минуточку, четыре "Гинденбурга". Конечно, взлетал он слабо заполненным, но на высоте раздувался на максимум, потому что воздух там ОЧЕНЬ разреженный. Даже по сравнению с потолком высотных самолётов. И стоит всё это дохуя, запуск обошёлся в $30млн, что уже сопоставимо с космическими запусками.

Стратостат на 10-15т (это вместе с ракетой очень маленький аппарат, кубсат практически, учитывая, что массовое совершенство у маленьких ракет хуёвое) будет стоить как трижды ёбаная в жопу оверпрайснутая Дельта IV хеви. Не дороговата ли экономия?
87 553359
>>3352
а хуй знает, я сейчас историю миссии читаю и её с таким трудом продавили. В один момент заморозили в буквальном смысле, забрали деньги, отменили, но потом восстановили, еще и пилили бюджет на ней знатно, присасываясь к ней чтобы передалать под солнечную обсерваторию, а потом отменяли эти планы, когда деньги уже были потрачены. Её вообще вице президент Эл Гор задумал, может камера и прочее оборудование с тех времён.
88 553362
>>3298

>прямо передо мной луна которая освящена лишь на половину


РПЦ не освящает невидимую половину.
89 553364
как будут выглядеть галактики и туманности вблизи? можно ли их увидеть своим глазом так же как и на фото, находясь от них на нужном расстоянии? как туманности выглядят изнутри? небо будет в узорах со всех сторон?
90 553380
>>3364

>как будут выглядеть галактики и туманности вблизи? можно ли их увидеть своим глазом так же как и на фото, находясь от них на нужном расстоянии?


Нет, конечно. Даже сам Млечный путь видно только при очень темном небе, а мы находимся прямо внутри него.

Если бы Андромеда была сильно ближе, ее (а не только сам яркий центр) было видно невооруженным глазом как туманное облачко наподобие Магеллановых облаков, но ничего похожего на фотографии с длинной выдержкой увидеть нельзя ни с какого расстояния.
91 553393
>>3380
я говорю про наблюдения с космического корабля а не земли
92 553398
>>3393
Разница невелика
93 553399
>>3362
Вода есть на обеих половинах, так что и освящаются обе
94 553400
>>3398
хорошо, но туманности то хоть немного похоже ирл видно будет?
95 553402
Может ли экипаж МКС заболеть уханьской простудой?
96 553404
>>3399
Чтобы заднюю половину освятить, нужно туда добраться, а там только протестанты и аметисты летали.
97 553406
>>3402
Конечно, какой-нибудь турист из Италии мимо проезжать будет, с пересадочкой, заразит там всех.
98 553409
>>3399
Задняя половина лунного диска недоступна для освящения. Да и вообще это философско-умозрительный концепт, ведь Луна прибита к небесной сфере, а "с той стороны" сферы может разве что Бог посмотреть. Может там и нет никакой второй половины.
99 553426
>>3404
Автохуй. Когда воду в водопроводе освящают, в трубы попа проталкивают штоле?
100 553430
>>3398
А если вылететь за пределы млечного и выключить свет на корабле, имея при этом девственно чистые глаза, не испорченные компьютером и вообще?
101 553432
Я раньше не задумывался, но загуглив обнаружил такой занимательный эффект как прецессия орбиты луны.
Крутится перегей, а так же независимо с большим сроком узлы. Наклонение я так полагаю статично. Вопрос как загуглить положение узлов и апо\перегея на данный момент?
102 553435
>>3432
Если тебе интересны цифры, то воспользуйся вот этим
https://ssd.jpl.nasa.gov/horizons.cgi
хотя там вроде нету узлов прецессии, но может я просто в глаза ебусь. Или тебе нужно что-то очень визуальное?
103 553436
>>3435
Ну визуализация никогда не повредит, но если этого нет то это хоть что-то.
Ух а это интерестная штука, не знал что такое есть. Спасибо.
104 553444
>>3436
>>3432
Если тебе прям точно прецессию посчитать - делаешь это сам, по актуальным эфемеридам согласно одной из моделей. В астрономии и космонавтике применяется три постоянно обновляемых модели движения тел Солнечной системы:
- JPLовская Development Ephemeris
- EPM от Института прикладной астрономии РАН
- INPOP, модель от Парижской обсерватории
Они немного разные, но расхождения минимальны, если тебе не научную работу писать или траектории АМС/спутников высчитывать детально. Они все учитывают довольно дохуя явлений, от релятивистских эффектов до возмущений от неравномерной гравитации. Опять же, ИПА и IMMCE ведут ежегодники, в которых есть готовые прецизионные расчеты, в том числе и прецессии, но они денех стоят.

Так-то институты ведут и обновляют согласно наблюдениям кучу моделей, о которых сразу не подозреваешь, но которые охренеть как широко применяются. NRLMSISE-00, например - актуальна и в астрономии, и в метеорологии, и в баллистике ИСЗ, и где только не. А обновляют её полтора постдока за еду. Фундаментальные сервисы они такие.
105 553447
>>3313

>А почему тебе кажется что луна всегда ровно светит мне неведомо. Возможно по той же причине по которой перевёрнутый вверх ногами или набок текст не осознаётся как перевёрнутый. И вообще это европоцентризм думать что сверху вниз, а не слева на право. Вот тебе флаг Мавритании с изображением луны.


Ну я никогда не видел чтоб луна была под углом. Почему-то в моем петербурге она всегда вертикальная.
106 553457
>>2819 (OP)
Насколько сильно фонит Земля на 50 герцах?
107 553458
>>3430
Твой глаз в любом случае не имеет такой выдержки чтоб цвета различать. Камерой увидишь, как и на земле. Глазом - никак и никогда.
108 553459
>>3457
Едва.
109 553584
>>3430
На пиках фотографический шоп фантазия автора скомпилированный из видимого, радио, ик, уф, рентгена и других диапазонов. В видимом свете вообще нихера кроме нескольких точек не будет видно, и скорее всего вообще без цветов.
110 553600
>>3457
Практически никак. Не говоря уже о том, что чем меньше частота, тем меньше фонит.
111 553621
>>3584
>>3600
не верю. что вам вообще об этом знать? вы это видели вообще???
112 553628
>>3457

>Насколько сильно фонит Земля на 50 герцах?


>


Вроде в ЛЭПах используется другая частота.
113 553637
поясните пожалуйста, если у вас есть космический аппарат для полёта к другой планете, и его двигатели вывели его на межпланетную переходную орбиту но еле-еле, буквально на десяток-другой метров в секунду скорость превышает вторую космическую. с какого момента такому космическому аппарату можно начинать тормозить чтобы перейти на орбиту планеты назначения если у этого аппарата допустим очень слабые движки ионники или вообще какой нибудь парус и ему нужно много времени чтобы затормозить?
114 553639
>>3637
Если еле-еле, то по большому счёту похуй. От любого чиха орбита сразу превратится в эллиптическую, и дальше уже можно будет вальяжно тормозить где удобнее. Гораздо хуже всё с быстрыми кораблями, с избыточной дельтой в 10-15км/с, а манёвр даже в упрощённой КСПшной механике длится часами.

Общий принцип такой:
1. От пизды ставишь точку манёвра, тянешь требуемую дельту в любом направлении и считаешь длительность манёвра.
2. Охуеваешь от длительности.
3. На траектории ставишь точку назначения, от считываешь от неё точку в 70-80% длительности манёвра. Нужен хороший такой запас.
4. Тормозишь вручную на глаз. Всё это время смотришь, куда смещается траектория, и подруливаешь еблом корабля, чтобы она более-менее оставалась на месте.
115 553643
>>3364>>3380

>ничего похожего на фотографии с длинной выдержкой увидеть нельзя ни с какого расстояния.


Есть один хинт.
Нужно съебать в тропические ебеня с нулевым световым загрязнением и упороть веществ, позаимствованных у шамана местных индейцев. Волшебные кактусы, вот это всё. Если всё сделано правильно, то зрачок расширится сверх нормы, а светочувствительность сетчатки возрастёт. Ну и мозг будет веселее обрабатывать информацию. Тёмные слабоконтрастные объекты будут не фильтроваться, а наоборот дорисовываться.
Плюс дополнительные приёмы с витамином А, красными очками, тренировками и.т.п. Можно очень пиздато забустить ночное зрение. http://lumenhouse.ru/articles/2851/
116 553645
>>3639
Лол, ебать я снайпер. Думал, это в огуречном треде вопрос.
Но ирл всё то же самое, только манёвры будут длиться не часами, а месяцами.
117 553647
>>3645

>Но ирл всё то же самое, только манёвры будут длиться не часами, а месяцами.


У тебя там двигатели Эпштейна чтоль есть, чтобы месяцами топливо жечь можно было?
118 553648
>>3645

к сожалению я ничего не понял из твоего ответа. разве если сразу начать гасить скорость то не получится так что если ты загасишь допустим эти 10-20м/с которые у тебя выше второй космической допустим за день или два с момента старта с орбиты земли, у тебя траектория твоей орбиты разве не замкнётся обратно вокруг земли?
119 553651
насколько вообще эллиптической может быть орбита вокруг земли в солнечной системе? чем это обусловлено? может какоето тело иметь орбиту вокруг земли в солнечной системе с самой дальней точкой в 1 а.е. от земли, например? может 10, 100, 1000? есть предел?
120 553652
>>3651
Сферу Хилла гугли.
121 553653
>>3652

>гугли



сложна
122 553660
>>3628
Нет
image.png31 Кб, 1200x638
123 553661
простейший способ сделать гору 100 км и в дальнейщем с ее помощью выходить на орбиту. пишу сюда спросить как назвать гору. я и так знаю что это сработает.
124 553662
>>3661
ах да, назвать думаю "атлант расправил плечи" вам как?
125 553664
>>3662

лучше "хмурый атлант одиночества"
126 553666
>>3664
я не под хмурым не надо тут свои смештучки суватьт!
127 553670
>>3661
Делай сразу до геостационарной орбиты, а то еще разгоняться надо.
128 553671
>>3670
А в центре горы ебануть лифт, и получится космический лифт.
129 553672
>>3670

>разгоняться



пушку поставим на вершине горы
130 553677
>>3661
Предлагаю ещё сделать верхушку горы плоской и соответственно саму гору расширить в несколько раз, чтобы было где хорошенько разогнаться. Потому что тебе надо не 100км вверх, а 7.9км/сек вбок.
Ой, а чего это земная кора вдруг провали
131 553678
>>3621
Да.
132 553681
>>3651
Орбита - это математическая условность, всего лишь прогноз траектории тела. Если ты этот прогноз тупо по школьному кеплеру будешь считать, то как только тело пересечет точку, в которой гравитация СС уравновешивается гравитацией соседних звездных систем и галактики в целом - оно обратно уже не вернется ни за какие шиши. Но тела летают не как Кеплер приказал, а туда, куда их тянет инерция, другие тела, и прочие внешние силы в данный момент.

Снаружи много звездных систем, они распределены неравномерно и тащат к себе по-разному, да еще и движутся постоянно. Поэтому полость гравитационного влияния Солнечной системы имеет неровную и постоянно меняющуюся границу. А тело с перицентром, почти упирающимся в край этой полости, может на одном витке вернуться назад, а на другом уже потеряться.

Во-вторых, по СС насквозь иногда летают всякие говна, которые могут повлиять на траекторию или захватить что-нибудь по пути, особенно если это что-то сейчас на самом краю гравитационного влияния Солнца и легко сталкивается на другую сторону. Вон например несколько десятков тысяч лет назад через Солнечную систему пролетала звезда Шольца.

Но в первом приближении можно сказать так. Сфера Хилла Солнца в отношении Альфы Центавра (наиболее близкая к нам сейчас) это примерно 2.3 световых года минимум. Поэтому если не тянуть апоцентр дальше примерно пары световых лет, назад тело скорее всего вернется, хоть и через огромное время, при условии что звезды не сдвинутся за это время (а они сдвинутся, и прилично).

Главное не забывать что 1) граница гравитационного влияния точно не известна, и 2) есть куча факторов кроме гравитации, такие как солнечный ветер, магнитное поле и т.п., которые вполне могут стать решающими на самом краешке полости, где гравитационное влияние незначительно.

>>3652
Сфера Хилла определена для пар тел. Эквипотенциальная граница Солнечной системы это сумма всех возможных сфер.
133 553682
>>3681
Мне кажется, он имел в виду что-то типа спутника Земли.
134 553683
>>3681
Блять, вот это я в шары ебусь!
В любом случае, это все применимо и к Земле - как только ты пересекаешь границу влияния, ты уже обратно не вернешься.
135 553684
136 553687
image.png91 Кб, 299x300
137 553692
>>3652
Бенни?
1584401108985.jpeg29 Кб, 240x320
138 553694
>>3692
Хэнка.
139 553699
Вот вы говорите гора 100 км провалится под кору. Насколько я понял верхние слои подминают жидкую астеносферу и не дают подняться выше. Но что если нагружать породу до тех пор, пока литосфера не встретится с нижней мантией? Будет ли тогда увеличен предел высоты предполагаемой горы?
140 553700

>нижняя мантия


твёрдая нижняя часть верхней мантии
>>3699
фикс
141 553701
двач я придумал вечный двигатель. берешь ионизированный газ, накачиваешь им куб посильней,магнит, и весь газ по магнитным линиям будет упорядоченно двигаться, тем самым давя только на 1 стенку, а значит создавать тягу вперед!! но все же мне думается, что то то подведет. что может подвести?
142 553705
>>3701
У тебя стационарное магнитное поле. Тягу ты какую-то получишь только при включении магнита, при изменении поля.
И нет, не получишь в закрытом объеме, поле и газ уравновесятся, постоянную тягу не будешь иметь.

>>3703 (Del)
>>3702 (Del)
Заебал бампать на слоудоске.
143 553709
>>3705
если это не возможно, тогда как летают летающие машины?
144 553711
>>3699
чево?
image.png275 Кб, 631x300
145 553712
>>3709
С помощью воздушных движителей, пропеллеров, например.
146 553713
>>3712
Это тачка?? это самолет...
image.png1 Мб, 1000x667
147 553714
>>3713
>>3709

>летающие машины


https://ru.wikipedia.org/wiki/Летающий_автомобиль
Taylor Aerocar это именно летающий автомобиль, все как заказывали.
image.png111 Кб, 275x183
148 553715
>>3714
вот летающая машина
149 553716
>>3715
Хватит троллить тупостью.
150 553717
>>3716
какой тупостью? самолет с крыльями. машина с колесами. летающая машина летает без крыльев, но с колесами
151 553718
>>3717
Такого не бывает, маня.
153 553724
>>3211

>Из-за постоянных ударов метеоритов образуются осколки и пыль



В голос!

>>3149
Неизвестно анон.
154 553725
>>3724

>В голос!


Какой голос? У тебя луна кратерами целиком покрыта, кратеры от чего на ней?
Больше так не тупи.
155 553726
>>3725
Постоянные == раз в сутки минимум

Мне чот слабо верится, что там дело только в метеорах и Солнечном свете.

Кстати, раз уж ты мне ответил. Если что-нибудь интересное почитать про ползающие камни и там и на Земле? Вообще есть что-нибудь не уровня вау шок, а простые факты, которые подтверждены?
156 553727
>>3726

>Мне чот слабо верится, что там дело только в метеорах и Солнечном свете.


Там миллионы лет это все проворачивалось, а не в одночасье.
Кратеры как раз и показывают, что эрозия весьма слабая, т.к. при сильной эрозии их бы замывало. Тем не менее - песочек реголит по сторонам раскидывает только в путь. Алсо термическое воздействие может обрушать стенки кратеров.
Вот и какая-никакая (почти никакая) эрозия.

>Если что-нибудь интересное почитать про ползающие камни и там и на Земле?


То?

>Вообще есть что-нибудь не уровня вау шок, а простые факты, которые подтверждены?


Нет, это зогач. Камни сами по себе не катаются.
157 553728
>>3727
А что советские учёные говорят по этому поводу?

И да, тебе же явно минимум 30 лет и ты ни разу не слышал про ползающие камни? Как так?
158 553729
>>3728

>А что советские учёные говорят по этому поводу?


Не интересовался.

>И да, тебе же явно минимум 30 лет и ты ни разу не слышал про ползающие камни? Как так?


Но ведь слышал.
Ты спросил "не уровня вау шок, а простые факты, которые подтверждены". Такого нет.
Тот факт, что тебя не устроил ответ подсказывает, что ты можешь быть сильно предвзят и ожидаешь подтверждения своим убеждениям. https://en.wikipedia.org/wiki/Confirmation_bias
159 553730
>>3729
Ты куда-то в сторону ушёл. Печально, что этот вопрос не изучали учёные, а ещё удивляются, почему их мало кто слушает.

>Не интересовался.



Вот это странно, они хотя бы точно туда луноходы отправляли.
160 553731
>>3730
Зачем заведомо шизоидный вопрос изучать?
Камни не двигаются, если их не двигать, это факт.
161 553732
>>3726
На земле есть ползающие камни, да. Это связанно с ветром, как конкретно перенаправляю тебя в гугл. На луне ветра нет, поэтому и ползающих камней там тоже нет.
162 553733
>>3732

>Это связанно с ветром


Со льдом.

>The movement of the rocks occurs when large ice sheets a few millimeters thick and floating in an ephemeral winter pond start to break up during sunny days. Frozen during cold winter nights, these thin floating ice panels are driven by wind and shove rocks at speeds up to 5 meters per minute.

163 553734
>>3731

А предположения, что это может быть некое природное явление, которое происходит после дождечка в четверг и с камнями, состоящими из кремния строго в диапазоне 60 - 70%, которые в этот особенный день особенно электризуются, так же важна поверхность, которая из-за влажной почвы и некого неизвестного эффекта.

>>3732

>Это связанно с ветром, как конкретно перенаправляю тебя в гугл



Лучше бы ты просто ссылку на видос закинул, в которой явно видно подтверждение таким заявлениям. Иначе как со всей наукой - подсчитали кароч, воооооот, двигаются ветром кароч, вооооот.

>>3733
А вот это уже интересно
164 553743
Аноны, а зачем люди сейчас занимаются освоением космоса? На космос тратятся огромные деньги, а выхлоп очень небольшой. Посудите сами: на то, чтобы "прогрессом" доставить партию трусов и водичьки для космонавтов тратятся деньги, на которые можно было бы построить несколько школ. На разработки тратятся огромные деньги, а в итоге всё, что нам доступно - улететь на несколько сотен км от земляшки, чтобы... чтобы нихуя. Люди осваивают космос уже почти 60 лет, но ни НЛО, ни дачи на луне - ничего.
165 553744
>>3743

>Аноны, а зачем люди сейчас занимаются освоением космоса?


Потому что больше деваться некуда, раз можем в космос, то будем осваивать космос, файнал фронтир, вот это всё.

>На космос тратятся огромные деньги


Да нет, не такие уж и огромные.

>а выхлоп очень небольшой.


Вот быдлу всегда сразу выхлоп подавай. Наукой занимаемся, за Землёй наблюдаем, за Солнцем, узнаём, что вообще творится, это очень важно. Не летать в космос - это как сидеть в шалаше в лесу и отказываться выглянуть наружу.

>тратятся деньги, на которые можно было бы построить несколько школ


В которых никто ничему не научится, если не будет яркого примера и мотивации, а космос как раз таким примером служит.

>улететь на несколько сотен км от земляшки, чтобы...


Проводить там эксперименты.
166 553745
>>3743
Я помню твою пасту, болезный.
На пасту ответ пастой:

Блять больше всего я ненавижу мудаков которые говорят , что космос не нужен.
Я всегда ссу на ебало этим уродам, которых к сожалению огромное количество. Сраные ебаные кретины, тупорылое быдло мечтающее о теплом угле, вкусной еде и мясной дырке. Неужели сложно понять простую вещь - Движение вперед это охуенно. Фронтир, неизведанное, полететь на Марс потому что можем и дальше.
И вот эти чмыри которые ненавидят космос, видят только унылое его использование приходят в спейсач и начинают кукарекать, что Марс не нужен.
Сдохните твари. Все что создано вокруг вас руками человека создано наперекор таким как вы. Все время существования человечества мы волоком тащим вас визжащих,упирающихся еблом по говну, камням и грязи вперед к прогрессу и таки протащили из обезьяны к запускающим в космос корабли разумным существам.
изображение.png558 Кб, 576x864
167 553748
Что делать, если я хочу быть таким охуенным как пикрел и строить ракеты, но я нищеинженер из рашки и мой максимум - РДТТ с потолком в жалкие ~20 км
168 553749
>>3748
Запускай огурцач, обмазывайся РСС и строй йобы скрывая свой бугурт.
изображение.png170 Кб, 640x360
169 553750
170 553751
>>3748

>РДТТ с потолком в жалкие ~20 км



Ракеты на твердом топливе и на орбиту выходят, за ними (а конкретно за японской SS-520) рекорд по минимальной массе орбитальной ракеты — 2,6 тонны.
171 553752
>>3748
Поступай в
>>499235 (OP)
И строй ракеты на говне
172 553753
>>3751

>на твердом топливе


смотря каком, если будешь активно интересоваться кое чем, то придет дядя майор и невежливо попросит пройти на бутылку
173 553754
>>3752
я уже, но что-то действительно стоящее построить требует оверкилл денег и усилий
174 553755
>>3748
Оцени свои финансовые возможности, и если их не хватает на обучение в США, то поступай куда-нибудь в Германию на бюджет, обучайся, работай, зарабатывай денег, продвигайся по службе, потом сможешь открыть свой космический бизнес, к тому времени уже возможно рынок услуг по снабжению Луны будет, можно делать лендеры, платформы всякие, роверы.
175 553756
>>3755
ТАм это самое, короновирус и вот это вот всё
176 553757
>>3756
Давай постараемся вскрыть эту тему. Ты молодой, шутливый, тебе все легко. Это не эбола и даже не грипп. Сюда можешь лезть. Серьёзно, только старперы и будут жалеть. Лучше не пытайся отмазываться и запомни что тут писалось. Я вполне понимаю, что данным сообщением не вызову дополнительного интереса, но сразу хочу подначить пытливых - вперед. Остальные просто не найдут.
177 553761
>>3754

>требует оверкилл усилий


Карьера пикрил - не для тебя
178 553762
>>3757
Неплохо.
179 553764
>>3762
Плохо
180 553766
>>3753
Полибутадиен запрещен, что ли, или перхлорат аммония, или порошок алюминиевый?

Это ж не экзотика типа фтор-мочи, такое твердое топливо жгут тысячами тонн каждый год.
181 553772
>>3748
Охуенность для обывателей. Будь суровым и неизвестным. Вкатывайся в безопорные технологии создания тяги.
182 553773
>>3772

>Вкатывайся в безопорные технологии создания тяги.


Это в зогаче изучением подобных технологий занимаются. У нас на доске все же больше про реально работающие технологии.
183 553776
>>3772

>Вкатывайся в безопорные технологии


Случайно прочитал "теологии". На секунду пост показался чуть более осмысленным.
sage 184 553778
>>3766

>


>Полибутадиен запрещен



Где ты его достанешь? А перхлорат аммония не то что бы запрещен, просто на карандаш могут поставить, ибо из него делают бризантные ВВ.
185 553779
>>3757
Вообще-то речь про то что европка закрыла границы. Да и из рашки вроде как не выпускают нынче. Поправьте если не прав.
186 553790
>>3778

>Где ты его достанешь


Да хоть прямо на али продают HTPB от 1 литра, это ж просто каучук, которым порошки склеивают в гранулы, чтобы они горели медленно, а не сразу по всей своей огромной поверхности, он применяется еще дохуя где.

Комбо перхлората аммония и алюминиевой пудры действительно звучит довольно стремно и может вызвать определенные вопросы, но законом не запрещено. Пока ты не начнешь их смешивать и заливать в формы, доебаться формально и не к чему, в закрытом гараже можно хоть шаттловский SRB построить.
187 553795
>>3778
И давно это из него делают бризантные ВВ?
>>3790

>законом не запрещено


Товарищи! Учитесь юриспруденции надлежащим образом!
188 553797
>>3744
>>3745
Понятно. Машем хуями в алкогольной яме дрочим вприсядку Исследуем космос! Усилий затрачиваем дохуя, а мкс в итоге выглядит как сраный гараж деда ебинея.
189 553798
>>3790
Как только вынесешь из гаража на всеобщее обозрение, у окружающих тут же возникнут вопросы, а ктонибудь очень сознательный и в мусарню позвонит. Разве что класть сразу в закрытый прицеп. И то гаишники могут прошмонать
190 553799
>>3790

> не к чему


П Р Е К У Р С О Р Ы
Р
Е
К
У
Р
С
О
Р
Ы
191 553800
>>3795
Покажи закон, запрещающий перхлорат аммония или алюминиевый порошок.

>>3799
Прекурсоры запрещены только у наркотических веществ, окислитель и топливо сами по себе не являются взрывчатыми веществами и полностью законны.
изображение.png483 Кб, 1061x696
192 553801
Хуле так дорого?
193 553802
>>3800

>


>Покажи закон, запрещающий перхлорат аммония или алюминиевый порошок.


Так никто не говорит, что закон запрещает. Просто на тебя внимание обратят. Поставят на карандаш. Ферштейн? А там до бутылки пол шага, учитывая реалии рашки.
194 553803
>>3801
По килограмму и более грубого помола всего 650 р/кг на этом сайте.
195 553804
>>3802
С таким настроем в рашке вообще ничего делать нельзя, а особенно сидеть на мейлрушной борде.
будь спок.jpg116 Кб, 300x387
196 553805
>>3798

>Как только вынесешь из гаража на всеобщее обозрение, у окружающих тут же возникнут вопросы, а ктонибудь очень сознательный и в мусарню позвонит. Разве что класть сразу в закрытый прицеп. И то гаишники могут прошмонать


Ну что вы как маленькие.
Называешь ракету ВЛАДИМИР ВЛАДИМИРОВИЧ ПУТИН-1, рисуешь на ней иконы, триколор, 1941-1945 можем повторить, портрет Кадырова и прочие атрибуты лояльности. Человек в погонах видит эти обереги и задумчиво икает, понимая, что если он помешает запуску ВЛАДИМИР ВЛАДИМИРОВИЧ ПУТИН-1, то премии от начальства не получит. А начальству не то чтобы не похуй, но страшно не выслужиться перед своим начальством. И так до бесконечности. Поэтому человек в погонах говорит "ну ладно" голосом Витали и делает вид, что его тут нет.

В ебанутом цирке важно не содержание, важна подача. С правильной подачей хоть ЯРД у себя в гараже запиливай.
197 553806
>>3803
всё равно дороха. 300р была бы норм ценой
198 553807
>>3804

>а особенно сидеть на мейлрушной борде.


Это можно, но крайне желательно с VPN
lol3.png7 Кб, 231x176
199 553809
200 553810
>>3801
Это не та ли хуйня, что охуительно взрывалась на складах с мощностью как небольшая атомная бомба? Че-то стремно такое дома хранить.
201 553812
>>3790

>Да хоть прямо на али продают HTPB от 1 литра


А ссылочку можно? Поискал, не нашел что-то.
202 553813
>>3810
Зато смерть будет быстрой.
203 553814
>>3813
И смерть еще половины дома.
204 553815
>>3812
Под али подразумевалась алибаба, а не алиэкспресс, всякие материалы типа пластиков, металлов и прочей хуйни именно там продают.
205 553827
>>3815
Как там покупать? Чёт не понятно
206 553828
>>3827
ПОСАН НЕ ПОКУПАЙ У МЕНЯ ТАК БРАТ ОТ КОРОНАВИРУСА УМЕР ХУЛИ ОН ПИЗДИТ ЧТО ЭТО БЕЗОПАСНО ПИДР??!? тебе не надо. Нафиг рисковать так-то?
207 553830
>>3828
Удваиваю этого, с топливами на основе перхлората аммония не шутят, можно и взорваться нахуй очень легко, и на бутылку присесть.

Если хочется чего-то запустить, бери проверенную тысячами ракетомоделистов карамель, импульс пожиже, зато живой будешь.
208 553831
>>3082
На полярные пофиг откуда.
209 553838
>>3701

>тем самым давя только на 1 стенку


Ошибка. Попробуй подучить физику.
210 553841
>>3699
Ты как не извращайся, не изменишь того факта, что земля в макро-масштабах представляет собой жидкое тело, поддерживающее свою форму исключительно самогравитацией. Что-то крупнее прыща на теле планеты уминается в эту форму, независимо от того из чего оно состоит.

Конкретно твоя хуйня была давно концептуализирована, только не в виде ебейшей горы, а в виде пусковой петли. https://en.wikipedia.org/wiki/Launch_loop
The Chain Fountain AKA The Mould Effect.webm1,7 Мб, webm,
720x404, 0:35
211 553842
>>3841
Как эта хуйня держится в воздухе.
212 553851
>>3830
>>3828
Чё бы ему взрываться? Не держи у открытого огня и ниче не взорвется.
213 553852
>>3743
Аноны, а зачем люди смотрят рекламу? На рекламу тратятся огромные деньги. Посудите сами: мало того, что стоимость рекламы включена в стоимость товара, то есть без рекламы товар был бы дешевле. На эти деньги можно было бы построить планету из школ, а маркетолухов сделать учителями в этих школах, а их знания по манипулированию и наебыванию, направить в полезное русло. Вдалбливаю пиздюкам с детства, что модно быть не тупой пиздой говноблогерром, а ученым мужем. Потому то чем круче твое открытие, тем чаще студенточки будут делать тебе минет между парами.
214 553855
>>3831
Хуй
215 553876
>>3810
На тех складах взорвалось, внезапно, около 4000ТОНН (4 ТЫСЯЧ ТОНН). Этот милый Антоша не сможет инициировать свой жалкий килограммчик даже на обычный пук-среньк.
216 553878
>>3852

>Аноны, а зачем люди смотрят рекламу? На рекламу тратятся огромные деньги. Посудите сами: мало того, что стоимость рекламы включена в стоимость товара, то есть без рекламы товар был бы дешевле. ...



а) Реклама следствие перепроизводства. Единственный способ ее убрать это сделать недопроизводство что бы люди сами разбирали любые калоши так как других нет.

б) Реклама "твой кампутер за год устарел, купи новый" кажется бессмысленной пока не меняешь временную шкалу на 10-20 лет. Внезапно выясняется что прогресс таки есть, и его двигают те хомячки которые ведутся на рекламу.

> Вдалбливаю пиздюкам с детства, что модно быть не тупой пиздой говноблогерром, а ученым мужем. ... студенточки будут делать тебе минет



Одно другому никак не мешает. Но пока силы говноблоггеров с ментача направлены на воспитание тезиса самок-сжеч-ряяя-нидают.
217 553883
>>3842
как?
218 553959
>>3851
В сухом виде взрывается от удара и трения.
219 553976
>>3807
Так через впн на мейлаче запрещено сидеть. Догадайся почему
220 553979
>>3976

>запрещено


А?
221 553998
>>3842
Момент же. Выдергивает с такой скоростью, что по инерции летит еще дальше вверх.
Вроде сам Молд и объяснял.
222 553999
>>3852
Было б пиздато.
223 554008
>>3804

>сидеть на мейлрушной борде.


Сидеть то можно где угодно, а вот перед тем как что-то писать надо думать головой.
224 554146
1. Если космонавт заболеет коронавирусом на МКС, его оставят там или отправят на землю?
2. Если нужно срочно вернуть МКСников на землю, они ждут ракету или у них всегда пристыкован модуль для срочной эвакуации?
225 554147
>>4146
1. Он не заболеет коронавирусом на МКС.
2. Всегда пристыкован.
226 554149
>>4146
как он заболеет-то, не контактирует ни с кем
227 554150
>>4149
Но ведь бактерии поднимаются до высоты МКС, вдруг в форточку залетит?
228 554157
>>4150
Пьяный астранафт решит покурить в форточку и засосёт бактерию, заражённую коронавирусом?
229 554163
>>4157
На ночь после перекура забудет форточку закрыть и ему надует вирус, да.
230 554164
>>4149
От компьютера.
231 554169
>>4146

>1. Если космонавт заболеет коронавирусом на МКС, его оставят там или отправят на землю?


Оставят там.
232 554171
>>4169
Чтоб Землю не заразил?
233 554175
Почему не запускают мелкие спутники с суперпушек?
234 554176
>>4175
Потому, что их распидорасит об воздух.
235 554177
>>4176
Почему бы не распидорасить мелкие спутники об воздух с суперпушек?
15495450211630.jpg65 Кб, 560x407
236 554182
>>4177
Действительно, почему бы и нет.
237 554184
>>4175
Не слышал о спутниках на суперпушках.
238 554187
>>4184
И я
239 554192
>>4175
Нет таких пушек, которые могли бы выдать требуемые 10+ км/с.
cd2[1].jpg14 Кб, 261x199
240 554198
>>3671
Что ты делаешь, содомит? Дома люди спят, а я ржу как конь
241 554200
>>3672
А почему нельзя катапульту поставить?
(Я спрашиваю серьезно)
242 554205
>>4200
Нужна двухступенчатая катапульта.
243 554209
>>4200
Так запускаемая часть один хуй сгорать на гиперзвуке будет.
Алсо нужна такая сила, чтобы не только давать ускорение грузу, но и преодолевать это трение атмосферы которая теперь еще и на плечо катапульты работает. Алсо жопа подсказывает, что таких материалов не завезли которые такую силу бы выдерживали.
244 554219
>>4200
>>3672
А что если разгонять ракету не пушкой или катапультой, а сделать так, чтобы она сама себя разгоняла?
245 554224
>>4219

>А что если разгонять ракету не пушкой или катапультой, а сделать так, чтобы она сама себя разгоняла?


Уходи, тебе тут не рады
image.png61 Кб, 299x168
246 554225
>>4219
ПИЗДОС. Ракета сама себя разгоняет!? Как тебе такая ахинея вообще в голову взбрела!?
Не, ну вы слышали!?
247 554264
>>4200
1. Катапультой нельзя запустить на орбиту, нужен хоть какой-то движок на запускаемом спутнике. Иначе запущенное либо ебнется обратно на Землю, либо улетит из сферы влияния Земли вообще, если хорошо запустить.

2. Сопротивление атмосферы. Тебе надо не 8 км/с достичь катапультой, а гораздо больше, потому что львиная доля импульса потеряется на сопротивлении атмосферы. А сопротивление растет примерно как квадрат скорости, ох щщи. Нагрев от такого сопротивления вряд ли что-то выдержит, не расплавившись.

3. Ускорение. Катапульта должна разгонять от нуля до 8-10-15 км/с на сравнительно коротком отрезке (размеры катапульты). Соответственно, ускорение должно быть чудовищным, мало что такое выдержит.
248 554265
Бывают ли атомные реакции без радиации? Чтоб четко продукты деления и хотя бы без ионизирующего излучения?
249 554267
>>4265

>атомные реакции без радиации


Радиация - это поток частиц. Чтобы реакция шла, она должна быть энергетически выгодной. Энергия идет на увеличение импульса частиц. Поэтому какой-то поток будет полюбасу.
250 554268
Насколько мощно надо заглубляться и насколько утягивать якорь будет космический лифт если мы его типа построим?
Нужны ли какие-то манипуляторы на земле чтобы подтягивать-подпускать "трос" на земле?
Можно ли сделать земную точку на плавучей йоба-платформе?
251 554269
>>4267
Да, я имел в виду радиацию в быдло-понимании. Ионизирующее излучение.
Можно ли иметь атомные реакции без ионизирующего излучения?
252 554270
>>4269
Вангну, что нет, но анус ставить не буду.
253 554271
>>4270
Мы хоть и на спейсаче, но мы не в спасех треде.

магнат анусов набитых на ставках в спасех треде
254 554272
>>4269
Нет. Хотя есть реакции поглощения нейтрона - в некоторых из них ионизирующего излучения не выделяется и они тоже ядерные. Но нейтрон должен быть низкоэнергетическим.
255 554273
>>4272
Вся фишка в энергетике этих реакций, да? Они преодолевают барьер и могут взаимодействовать с ядрами?
256 554281
>>4171
Чтоб не отправлять экипаж домой раньше срока почём зря, и чтоб не заражать других членов экипажа. Там его изолируют, будет на карантине. Не знаю, есть ли у них аппараты ИВЛ, может и есть, но в любом случае, предполагается, что космонавты ребята крепкие, и корону смогут пережить без осложнений. Но это конечно мои спекуляции, реально я не знаю их протоколов.
257 554282
Когда на марсе будем, хлопцi?
image.png73 Кб, 220x220
258 554283
Насколько сильно надо переделать двигатели обычного реактивного самолета чтобы он мог невозбранно летать на Титане заправленный кислородом или тетраоксидом (без учета замены на коррозийно устойчивые деталей топливопровода)? Давление, подачу, что-то поменять? Может структура пламени заставить двигатель делать иной формы?

Вопрос 2 - что будет являться наиболее удобным источником энергии на поверхности Титана?
259 554284
>>4282
Схоняй в махаз, возьми батончик, да присядь, хлопец. Бац! Ты сел на марс!
Не раньше 2030, ничего раньше не светит для людей, никаких нормальных тем. Если КНР и США начнут космическую гонку вотпрямщас - то до 2030 может кто и сядет. А так - хуй там был.
Не факт что на луне-то до 2030 будет человек. Возможно, но я не сильно в это верю.
260 554285
>>4284
Грустно как-то! Еще эта панда мия появилась блин...
261 554286
>>4284
Да даже если начнут сейчас гонку, максимум к середине 30-х осилят. А на Луне наверное будут, почему нет. В 24 вот не факт, но думаю не сильно позже.
262 554287
>>4286
А они астронавтов уже сейчас готовить начали?
263 554288
>>4287
Для Луны? Да.

Кстати, вот мои вангования: до 2026 облёт людьми Луны, до 2028 высадка. Старшип полетит на орбиту в 2022, с людьми до 2030, тогда же и к Луне. На Марс с людьми в начале 40-х.
264 554289
>>4288
Выглидт адекватно. Надеюсь на такой расклад. На свое 40 летие увижу людей на марсе.
265 554290
>>4285

>Еще эта панда мия появилась блин


Тот же вирус и в 2003 такой же бугурт вызывал. Человечество, как вы все помните, вымерло с концами. Конец света случился, и нас с вами не стало.
Самое печальное, что ни один из концов света так и не научил пиздоглазых не жрать мышей и масло из канализации, не срать на улице (да, они тоже), и мыть руки.
Чихай в локоть, не трогай лицо, мой руки - так живи и все будет нормально.

>>4286

>Да даже если начнут сейчас гонку, максимум к середине 30-х осилят. А на Луне наверное будут, почему нет. В 24 вот не факт, но думаю не сильно позже.


С китайцами? Может и проиграют. Эти могут не жалеть жизни тайконавтов и ускорить темпы. Если вдруг такой манямирок случится писькомерства, разумеется.
Я потому и писал - если. Как и раньше - к 2050 может быть и будет что. В ближайшую декаду ничего не намечается реального.
266 554291
>>4290

>Тот же вирус и в 2003 такой же бугурт вызывал.


Я не про опасность для ЧИЛОВИЧИСТВО, я просто про панику и экономические тормоза, которые замедлили ход истории (history (Geschichte)).

>Самое печальное, что ни один из концов света так и не научил пиздоглазых не жрать мышей и масло из канализации, не срать на улице (да, они тоже), и мыть руки.


Ты что, это же их КУЛЬТУРА.

> В ближайшую декаду ничего не намечается реального.


А луна?
267 554292
>>4291
Пфф, это не вирус замедлил, это я сама замедлила, на лыжах.
Паникеры были и тогда.
Просто поток пиздоглазых калосёров в те годы был не такой активный, вот тот штамм коронавируса тогда и имел меньшее распространение.

>А луна?


Я про высадку на Марс говорил.
С Луной... Не думаю. 20% что будет. 2028 ближайший реальный срок и скорей всего словит перенос.
268 554294
>>4293 (Del)
Ты промахнулся, да?
269 554295
>>4293 (Del)
Хуита.
Рассказ о том что вся вселенная это симуляция студента говнокодера, которому лень было ебашить реалистичную гравитацию и он её задал константой лучше.
Там в итоге люди выстроили из галактик надпись на весь монитор ГОВНОКОД
270 554296
>>4295

>выстроили


Лол, продолжение что ли есть? Я думал просто говнопаста.
271 554297
Поисните, как у Сагана в "Контакте" Б-г умудрился оставить послание в числе пи? Ведь это математическая константа, не зависящая ни от чего вообще. Даже если бы вселенная была пиздец кривая как у Лобачевского, пи (как отношение длины окружности к диаметру в умозрительной евклидовой геометрии) было бы ровно то же самое. Тождество Эйлера, опять же.
272 554298
>>4283

>Насколько сильно надо переделать двигатели обычного реактивного самолета чтобы он мог невозбранно летать на Титане


Чуть более чем полностью.

>Вопрос 2 - что будет являться наиболее удобным источником энергии на поверхности Титана?


Ветряки вряд ли, так что ритэги.
273 554299
>>4298

>Чуть более чем полностью.


Я вот чувствовал, что разница должна быть от подачи топлива в окислительную смесь от подачи окислителя в топливную, но я не знаю какая.

>Ветряки вряд ли, так что ритэги.


А почему не сразу реакторы? Теплообмен должен позволять.
274 554300
Не может быть такого, чтобы такой большой космос был недоступен для исследований. Во всех фантастических мирах типа экспансии или масс эффекта люди находят нечто, что открывает возможности для экспансии космоса. Значит и ирл что-то такое найдут, новый химический элемент, вещество какое то или ещё что. Просто надо тщательно обследовать свою систему, все астероиды и все планеты. Им миллиарды лет, некоторые вообще прибыли из глубин космоса, где то да найдётся что то эдакое.
275 554301
>>4300

>Значит


Это ты так примерно почувствовал?
276 554302
>>4301
Да.
277 554304
>>4300
а-а, понятно, спасибо что рассказал
278 554305
>>4299

>А почему не сразу реакторы?


Ритэги компактнее и легче, только и всего. Если есть возможность привезти целый реактор, то почему бы и нет.
279 554309
>>4264
>>4209

А если придумать катапульту-стульчак? То есть, запускать со стульчака-катапульты капсулу, "срущую" огнём (ну на каком там топливе взлетают космонавты)? Ну, то есть, как бы объяснить-то... Есть катапульта в форме стульчака (круг с дыркой). Ставим ракету жопой, из которой издаётся пламя, на этот "стульчак". Запускаем катапульту, ну а потом включаем двигатели, ракета ебашит в космос... профит!!!! Экономия топлива+новые ощущения+нанотехнологии.
280 554314
>>4305
По частям нельзя привезти? Да и есть же небольшие реакторы. Вон в подлодках же ставят.
281 554315
Я не понел, почему катапульту нельзя поднять на аэростатах на 20 км над землёй и запускать оттудова? Алсо запускать можно рейлганами. Вон учёные достигли скорости снаряда в 8 км/с. Это больше первой космической.
282 554317
>>4273
Кто они-то блядь?
283 554318
>>4295
Не было там про кривую гравитацию
284 554331
>>4300
Нет, не найдется. И скажи спасибо, что мы хоть в телескоп можем увидеть что происходило в прошлом. Мир не сахар, никто ничего тебе не должен. Даже с ядерным синтезом вполне может оказаться, что человечество не сможет его использовать, а уж про полеты к звездам за человеческую жизнь лучше вообще не думать
285 554332
>>4331

>Даже с ядерным синтезом вполне может оказаться, что человечество не сможет его использовать


Но оно уже полвека как может.
286 554336
Может ли быть так чтобы на планете острова были как грибы - с узкой ножкой и широким всем остальным? Необязательно земляные грибы в воде, а вообще в целом?
287 554340
>>4332
да и гравитацию ещё до нашей эры использовали
чё там пиздят что не погут её объяснить
288 554341
>>4340
Могут же. Уже лет пятсот как могут. Каждый раз точнее, но от изменения объяснения принципы не менются, все работает.
289 554346
>>4336
Да, если они изо льда
image.png6 Кб, 366x243
290 554347
>>4346
Вопрос о том, чтобы ножки касались дна, чтобы внизу было твердое, чтобы вокруг жидкость и грибовидный остров. Пикрил.
291 554350
>>4341
Объяснить и описать две большие разницы.
292 554351
>>4350
Работает? Работает. Предсказательную силу имеет? Имеет.
Чего ты ещё хочешь-то?
Некоторые вещи могут быть непостижимы с наскоку обычным мясным мозгом который вне мира который изучает.
Кто может чествно похвастать, что ориентируется в 4д пространстве? (Я не про рубик). Кто может похвастать, что понял квантомех на интуитивном уровне?
293 554352
>>4347
Есть такая штука дифференциация вещества. Тяжёлое вещество будет стремится вниз, а лёгкое вверх. Не забываем о сопромате, который не даёт образовываться вертикальным структурам из природных материалов выше определённого предела. 15 км для земли например. Подобная структура, если и возможна, то либо на маленьких планетах с не очень большой гравитацией, тогда и жидкость будет явно не вода, а какая-нибудь специфичная штука, что не улетит в космос. Естественно-природно такое образоваться явно не могло. Что на астероиде н-километров было, что-то жидкое занимающее больше 80% объёма. Это уж какая-то дичь. Но если к примеру сделать ножку покороче, эдак на 3 порядка, и пошире раза 4 то сопромат вроде позволит. Но и тогда должны быть определённые условия существующие 1 раз в несколько кластеров сверхскоплений галактик.
294 554353
>>4352
А ножка и не обязана быть несущей. Более того, она могла бы быть плавающей, отсутствующей.
По-моему, Пратчетт что-то такое писал, нет?
295 554354
>>4351
"Понимание", как и "восприятие" с "интуицией" вообще-то субъективное понятие. Квант мех ими не оперирует.
Описание действительно описывает и действительно если ты прочитаешь про тигра, что у него есть полосы ты действительно придя в зоопарк их увидишь. Эволюционное объяснение же этих полос будет кардинально отличатся.
296 554355
>>4353
Насколько я знаю Пратчетт писал фентези про ведьм и смерть с косой. Если ножка зависнет, это значит что она имеет такую же плотность как окружающая её жидкость. И даже если эта ножка состоит из сверхпрочного материала ближайшее небесное тело своими приливными силами разрушит этот материал в щепки(ну допустим километровые щепки) И они в свою очередь плавно растекутся по всей сфере на которой они имеют такую же плотность как и окружающая их среда.
297 554356
>>4354
Развей свою мысль, пожалуйста. Пока что ты мне ничего нового не сказал.
298 554358
>>4356
Речь изначально была о том, что мы гравитацию описали, но не знаем как её по нормальному использовать ибо не знаем как она устроена. А потом ты что-то про интуицию начал загибать, и я повторил тезис, что объяснение и описание разные вещи. Поясни какую мысль я должен развить?
299 554359
>>4358

>но не знаем как её по нормальному использовать ибо не знаем как она устроена


Это не так. Мы имеем хорошее представление как она устроена и используем её нормально.

> потом ты что-то про интуицию начал загибать,


Про вещи которые с тем как окружение устроено не вяжутся.
300 554369
>>4359
Ты хочешь сказать ты 100 % уверенно знаешь какие процессы протекают чтобы гравитирующий объект гравитировал?
Какие вещи не вяжутся с окружением? Чьим окружением? Как не вяжутся?
301 554404
>>4369

>Ты хочешь сказать ты 100 % уверенно знаешь какие процессы протекают чтобы гравитирующий объект гравитировал?


Разумеется, нет. Наука так не работает.
Я знаю что в 100% случаев на земляшке и близ нее буду иметь ону траекторию.

>Какие вещи не вяжутся с окружением? Чьим окружением? Как не вяжутся?


Никакие. С реальностью не вяжутся.
302 554424
>>4314

>Вон в подлодках же ставят.


Мне лень гуглить массовогабаритные параметры реакторов апл, но там явно счёт идёт на тонны и метры. А у ритегов на килограммы и метры. Но тут вопрос в том, какая нужна мощность. Если нужны мегаватты, то юзать ритеги действительно глупо.
303 554432
>>4314

>По частям нельзя привезти?


Нет. Все космические реакторы запускали целиком. И устанавливают реакторы только на заводах
304 554445
Почему наливание кипятка звучит как-то мягче, иначе, нежели наливание не горячей воды?
305 554450
Какие животные лучше всего себя чувствуют в невесомости?
Птицы почему-то вообще нихуя не могут в нее, вроде бы.
306 554454
Анон, вот ходили пару лет назад слухи о планах отправить к центавре армию нано-спутников. Интересно, какой начинкой современная наука может снабдить такие аппараты и какие данные о другой системе эта начинка может в теории собрать и насколько реальна их передача из другой системы обратно на Землю ?
307 554455
>>4454
Обычным консюмер-грейдом в космическом исполнении.
Когда спамишь - можно забить на redundancy, оно у тебя в числе аппаратов заложено.
Передавать будет 4 года, когда и если долетит. Передаст инфу об атмосфере на планетах.
308 554456
>>4450
Чем сложнее устроен организм и больше приспособлен к конкретной среде, тем хуже.
Бактерии проще и способны жить везде им похуй.
На насекомого и так хуй какая гравитация действует.
Мышка всё же норное животное, плавать как космонавты не сможет, но в трубах-террариуме вполне обитают.
Птицы мало того что сложноустроены, так ещё и специалисты противодействия гравитации. Гравитации нет механизмы работают в холостую и им хуёво.
Люди как и любое среднеразмерное животное будет чувствовать себя хуёво, но многие косяки сглаживает универсальность и возможность жить в любых средах за счёт интелекта. У животного с ёба инстинктами, развилась бы паника и разрыв сердца от охуевания.
309 554457
>>4456

>У животного с ёба инстинктами, развилась бы паника и разрыв сердца от охуевания.


Достаточно продвинутый моск как у хомосапенса может корректировать восприятие. Ты падаешь. Но ты достаточно долго падаешь чтобы ты мог собраться с мыслями и что-то делать.
310 554459
>>4455

>консюмер-грейдом в космическом исполнении


Чиво блядь?
311 554462
>>4459
КОНСЮМЕР ГРЕЙДОМ В КОСМИЧЕСКОМ ИСПОЛНЕНИИ
Конкретнее - не специализированные жирные рад-защищенные йобы, а более-менее обычные процы с РТОСом на борту в трипликации. Дешево и многие из них помрут. Но дешево.
312 554470
>>4457
я не падаю
313 554473
>>4470
Это поправимо.
314 554484
>>4450

>Какие животные лучше всего себя чувствуют в невесомости?


Бамп вопросу.
.jpg151 Кб, 768x768
315 554485
>>4300

>Значит и ирл что-то такое найдут


Хуле тут искать? Все уже найдено до нас.
image.png298 Кб, 480x362
316 554486
>>4485
Но мы пока не видели Лилуминаии Лекатариба Ламина-Тчай Экбат де Себат.
317 554487
>>4486
Вы просто не обращаете внимания.
image.png149 Кб, 680x680
318 554488
>>4487
Обращаю. Пятый элемент продается практически в каждом продуктовом этойстраны.
image.png137 Кб, 373x210
319 554489
>>4488
>>4487
Пик не рилейтед, кстати, я говорил про бор.
320 554493
>>4484
>>4450
Плохо чувствуют себя те, кто умеет летать, по понятным причинам. Ещё вроде бы кошки плохо переносят, опять же, потому что у них есть программа на случай падения, и вообще они сильно полагаются на вестибулярку, которая там не работает. Лучше всего переносят рыбы, насекомые, одноклеточные.
321 554494
>>4493

>Лучше всего переносят рыбы, насекомые, одноклеточные.


Доставьте инфу по рыбам!
Как у них плавучесть работает-то?
322 554495
>>4494

>Как у них плавучесть работает-то?


Как и на Земле, епта, рыбы имеют нейтральную плавучесть из-за того, что их средняя плотность равна плотности воды, и это соотношение сохраняется при любом ускорении свободного падения. От внезапного пропадания гравитации плавать они не перестанут.
323 554496
>>4495
Эээ... Они ориентируются благодаря плавательному пузырю, который их ориентирует вверх.
Без него они плавают без ориентации.
И другое - я нихуя не могу по рыбам в космосе нормальное найти.
image.png75 Кб, 500x327
325 554498
>>4497
Вот я дегенерат. Мог бы и сам найти.
Спасибо, анон, добра и здоровья тебе.
326 554499
>>4497

>А. А. Леонов в бортжурнале сделал запись[25]:


>«Как себя чувствуют рыбки? -Хорошо, они все погибли»



Бесчуственные совки!
image.png279 Кб, 640x476
327 554501
>>4499
Или наоборот. Сочувствовали и поняли что им лучше погибнуть чем дальше страдать.
328 554504
Народ, вы видели Ланиакею?
вы, сука, понимаете, что каждый сраный атом в видимой вселенной встроен в гравитационную структуру, которая падает на два великий аттрактора?
как у вас пархатый язык поворачивается кукарекать про расширение вселенной после этого?
329 554505
>>4504
Лол.
У тебя с ложки суп упал вниз на тарелку. ЗНАЧИТ ВСЕ ПАДАЕТ В ЭТОМ НАПРАВЛЕНИИ.
Когда как в реальности тебя и твой суп все время тащило к солнцу. А ты не замечал.
И сейчас ты приходишь, отрицаешь все наблюдения и говоришь, что все летит к Аттрактору, забыв про объективную реальность.
330 554506
>>4450
Черви-пидоры и простейшие
331 554507
>>4505

>отрицаешь


какую нахуй объективную реальность?
332 554508
>>4462
А космическое исполнение по-твоему в тернировании?
333 554510
>>4504
А ничего, что мы улетаем от аттрактора в десять раз быстрее, чем падаем на него?
334 554511
>>4510
Если б мы улетали от него, его не называли бы аттрактором. Полетим мы быстро, решительно не скоро.
335 554513
>>4506
А самые сложные кто?
Инб4 люди

>>4508
Нет. Не только, но оно очень важно.
336 554518
>>4513
Всё-таки индустриальное, военное и космические исполнения - это про жырноту транзисторов, а не кратность дублирования. Электронику итак дублируют. Правда, не в одном чипе.
Моё ИМХО - нечего экономить на спичках, процы составляют не такую уж большую долю стоимости научных аппаратов, а корректная их работа критична. Правда, можно съэкономить на энергопотреблении, но тут просто хуй к носу не прикинешь.
337 554519
>>4518

>съэкономить


Ну ебаный ты помидор. Анон, ей-богу.
338 554521
>>4510
тогда бы гравитационная структура развалилась бы миллиарды лет назад
а она существует потому что Аттрактор структурирует материю так же как ЦЧД структурирует галактики
все манятеории давно пора заново пересматривать
339 554522
>>4511

>Полетим мы быстро, решительно не скоро


когда остатки нашей системы будут наматываться на ближайшую ЧД
340 554523
>>4504

>аждый сраный атом в видимой вселенной встроен в гравитационную структуру, которая падает на два великий аттрактора


Боюсь тебя расстраивать, но гравитация не настолько дальнодействующая.
341 554524
>>4519
Блядь, ну смотрите, чему меня тут научили
342 554550
>>4523
объективные данные про Ланиакею говорят обратное
кого мне слушать? ученых астрономов или твой вскукарек на Дваче?
343 554560
>>4499
>>4501
Разве это не метафора? Дескать на том свете им хорошо.
344 554562
Если все что мы на небе доходит до нас с большим пингом то почему мы видим падающие звезды со скоростью света ирл без задержки?
345 554563
>>4562
Задержка все равно есть. Ну и это никакие не звезды, а микрометеориты, до которых от наблюдателя от силы сотня-другая километров, и пинг там идет на микросекунды.
346 554602
Как можно получать электроэнергию, какие способы?
Градиент температур с помощью кипятильника ядерного или химического.
Фотовольтаика.
Есть ли какие-то хитровыебанные способы которые из-за сложности или неудобства не юзают, а могли бы?
347 554607
>>4602
Можно получать электроэнергию из людей, при этом подключив их к виртуальному миру, чтобы они думали, что они живут как обычные люди, ходят на работу и так далее.
image.png1,2 Мб, 1200x680
348 554608
>>4607
Пробовали уже, полная хуйня выходит.
349 554614
>>4602
Это ты способы преобразования описал, а не источники. Градиент температур обычно преобразуют в электричество через пар, механику и электромагниты, а не напрямую.

Если тебя интересуют готовые аппараты (источник+преобразователь) применительно к космосу, есть например ядерно-оптические преобразователи, например. Сумрачная идея какого-то злого гения из ВНИИЭФ. Частицы делящегося материала подвешены в газе под давлением 100 атмосфер, и возбуждают его атомы распадом, тот начинает светиться. Поскольку это практически монохромное излучение - можно его преобразовывать фотовольтаикой с огромным КПД. Плюс в том, что ты получаешь охуенный источник электричества, по плотности на кг сравнимый с баком бензина, что бесценно для космического аппарата. И предельно простой и не имеющий термодинамических и сложностных проблем реактора при этом, т.к. реакция не идёт. Минус в том, что это натурально грязная бомба. Бак с радиоактивным говном под 100 атмосферами, малейшая протечка в атмосфере и мало не покажется никому. А если ебнет в космосе, лишишься источника энергии (и возможно аппарат распидорасит). Даже если просто стравит, будет менять траекторию реактивной струей. В отличие от РИТЭГа, которому пробоина от маслины на орбите сравнительно похуй.
350 554622
>>4614
Впервые слышу, как называется эта штука?
351 554625
>>4622
ЯОП, ядерно-оптический преобразователь. Концепт, который давно известен, но вяло исследуют, ибо перспектив мало. Что-то говорили про переработку ОЯТ и космическое применение, но это болтовня, слишком опасная штука.
352 554627
>>2819 (OP)
Что будет с моим писюньчиком, если я находясь на орбите черной дыры, попробую его засунуть за горизонт событий?
353 554628
Чо там с двигателем из микроволновки? Тяга есть?))
Снимок.PNG521 Кб, 950x694
354 554631
>>2819 (OP)
Почему я данные не могу передать на кербал?)
355 554632
>>4628
После того, как на земле продемонстрировали, что направление тяги никак не зависит от ориентации ведра, уже было понятно, что это никакой не двигатель. Нет, тяги нет.
356 554633
>>4627
Никто не знает, но можешь попробовать во имя науки, я разрешаю. Правда ты тоже не узнаешь, тебя спагеттифицирует задолго до горизонта

>>4628
Нет
357 554634
>>4633

>тебя спагеттифицирует задолго до горизонта


Врети
15836112398530.jpg54 Кб, 640x491
358 554637
Хммм...
Чоб спросить такого эдакого, у ученых анонят няш-умняш.
Вот в ютубах показывают, что мол БВ произошел в одной точке. Но так ли это? Как обстоят дела на самом деле? Я вот слышал, что как бы везде бахнуло одновременно.
359 554638
>>4637
Сингулярности не существует.
360 554639
>>4638
Это как? А пространство бесконечно или замкнуто само на себя? А вещество в ней?
361 554641
>>4627
Ну, во-первых, с орбиты ты своим писюном до горизонта событий не дотянешься, потому что стабильные орбиты заканчиваются еще задолго до самого горизонта, на 3/2 шварцшильдовского радиуса.

Во-вторых, горизонт это не какая-то осязаемая поверхность ЧД, никакой видимой границы там нет, и засовывать хуй тоже по факту и некуда.
Это просто воображаемая поверхность, с одной стороны которой вылететь наружу из ЧД еще теоретически возможно, хотя для этого и потребуется фантастическое ускорение, а с другой не поможет уже вообще ничего — там для вылета из черной дыры потребуется двигаться быстрее скорости света, что законы физики запрещают.

В-третьих, если тебя не смущает все выше написанное, и ты все же решил пожертвовать писюном во имя науки, то засунуть-то его под горизонт ты сможешь без труда, как и запрыгнуть туда самостоятельно, а вот обратно высунуть — уже нет.
362 554642
>>4641

>а вот обратно высунуть — уже нет


Ну вот залупку я за горизонт все таки умудрился засунуть руками. Что произойдет, когда я попробую его вытащить обратно?
363 554643
>>4641
Кстати, раз уж такая хуйня. Значит ли это, что человеческий организм разладится и умрет на орбите вокруг черной дыры, даже сверхмассивной (приливные силы минимальны)? Ведь есть, например, кровообращение, да вообще вся хуйня, обязательная для жизнедеятельности. Если за горизонтом событий нельзя двигаться "наружу" вообще, то вблизи, но снаружи, это тяжелее или нет?
364 554644
>>4637
Если взять за основу хаббловскую модель расширения, то всё вокруг. Но не бахнуло, а разуплотнилось. То есть 14ккк лет назад кругом был ад пизды пожарче и плотнее чем в любом солнце, а затем расширялось само пространство. Этот вывод делается на основе того, что скорость расширения пространства всегда была более-менее одинаковой, за исключением первых наносекунд, где она была сильно быстрее. Однако если у расширения пространства есть ускорение, вся модель идёт по пизде.
365 554645
>>4633

>тебя спагеттифицирует задолго до горизонта


Это зависит от размера ЧД
366 554647
>>4642
Около горизонта ты будешь двигаться в околосветовой скоростью, соответственно для тебя время замедлится почти до нуля. Действия по вытаскиванию залупы для тебя будут равны миллиардам лет для чд, за которые ты давно провалишься сквозь горизонт
367 554648
Прохуялишься. Я рядом со своим членом на устойчивой орбите нахожусь. Начинаю свой свисток погружать за горизон событий, ради науки. Что происходит в этот момент? Что я почувстую?
368 554649
>>4648
ты умрёшь
369 554650
>>4642
Атомы хуя под горизонтом расцепятся с атомами хуя, находящимися над ним и он порвется.

Т.к. внутри черной дыры никакое взаимодействие не может распространяться в направлении изнутри наружу, межатомные силы просто перестанут действовать. Это ж просто электромагнитное взаимодействие, переносимое фотонами, без фотонов не будет и связи между атомами.
15839550308640.jpg34 Кб, 417x350
370 554651
>>4650

>электромагнитное взаимодействие, переносимое фотонами


Лишь фотоны могут что то там переносить? А вдруг есть частицы, которые могут на хую вертеть законы относительности нагибая все известные нам ограничения, и переносить энергию в подпространстве)
371 554652
>>4643
Орбит там нет. Вблизи к горизонту событий требуется постоянно ускоряться в направлении наружу от него, чтобы не упасть под горизонт, и чем ближе, тем большее требуется ускорение. Самое близкое расстояние, на котором можно обращаться вокруг ЧД с выключенными двигателями — 3 шваркшильдовских радиуса, ближе этого расстояния стабильных орбит не существует.

https://en.wikipedia.org/wiki/Innermost_stable_circular_orbit
https://en.wikipedia.org/wiki/Photon_sphere

Скорее всего, тебя размажет в паштет еще задолго до самого горизонта, когда мышцы и кости больше не смогут выдерживать перегрузки от работающего двигателя.
372 554653
>>4651

>А вдруг есть частицы, которые могут на хую вертеть законы относительности нагибая все известные нам ограничения



Решил украсть мою нобелевку за открытие сверхсветового движения, пидор? Аааа, хуй тебе!
373 554655
>>4653

>сверхсветовое движение


А ты допускаешь подобное? Расскажи почему. Оч интересно
374 554661
>>4631
Антенна где?
375 554662
>>4652
Не обязательно орбита прям впритык. Пусть будет 3 радиуса. Вопрос вот в чем: если тело вращается вокруг СМЧД на стабильной орбите достаточно близко к ЧД, будут ли какие-то трудности в перемещении "внутренностей" по телу, то есть в направлении от дыры? Именно из-за искривления пространства, а не из-за приливных сил (считаем, что ЧД достаточно СМ, чтобы они были маленькими). Например, с кровообращением.
376 554664
>>4662
Да вроде нет, проблемы могут быть только из-за приливных сил, а вернее, из-за разницы в силе гравитации между, скажем, головой и ногами. На преодоление этой разницы сердцу нужно будет затратить определенную работу, чтобы поднять кровь из более глубокого гравитационного колодца, но просто так сама гравитация никак не будет ощущаться.
377 554677
Видел сайт давно, где размер солнечной системы не условный с большими планетами в несколько диаметров друг от друга, умещающийся в размер картинки, а показывался реальный масштаб, Луна размером с пиксель и можно было скроллить колесом мыши до посинения полчаса от Солнца до Нептунов всяких.
Нет ни у кого, а то найти не могу, может его и нет уже? Или подобное что-нибудь.
378 554686
>>4644
понятие пространства времени - попрежнему спекуляция основанная на триллионные долях процента которые происходят из-за того что рядом просрался Джон
379 554693
>>4677
Легко ищется по запросу «moon 1 pixel».
https://joshworth.com/dev/pixelspace/pixelspace_solarsystem.html
14598086320020.png20 Кб, 317x397
380 554702
Поясните за орбиту. Почему это возможно в огурцах? И почему когда я давно и пытался повторить орбиту в огурцах у меня получалась хуйня, более близкая к реальности, чем эта?
381 554703
>>4702
Это и ИРЛ возможно.
Манёвр Оберта работает.
382 554706
>>4627
Эх, а меня лет 5-6 назад ровно за этот же вопрос забанили.
383 554707
>>4706
Так лучше же. Тебе стало сразу понятнее, что не надо пиструн в черную дыру пихать. А вот тот анон может и не догадаться без бана и попробует.
384 554708
>>4703
Насколько я понял подвох в том что компановка спутников редка Так?
385 554709
>>4708
Щито.
386 554711
>>4707
Я решил что астрономы скрывают секреты межгалактической дрочки и уже на пути к А Стрельца.
image.png42 Кб, 180x251
387 554712
>>4711

>межгалактической


>дрочить с помощью СМЧД своей же галактики


Значение знаешь?
388 554713
>>4712

> своей же галактики


Не стоит вскрывать эту тему
389 554714
>>4651
А вдруг тебя каждую ночь ебут гномы-ниггеры пока ты спишь|?
390 554715
>>4714
Очень надеюсь на это.
391 554718
>>4644

>Однако если у расширения пространства есть ускорение, вся модель идёт по пизде.


С чего бы это?
392 554719
>>4718
Так манямирок не рушится.
393 554733
я понял как терроформировать марс лол
394 554734
>>4693
Спасибо.
395 554735
Ребятки, как измеряется гравитация в открытом космосе7 Как уловить разницу в гравитации на орбите земляшки и на орбите урана7 В каких единицах измеряется7
396 554740
>>4735

>В каких единицах измеряется7


В ньютонах.
>>4735

>Ребятки, как измеряется гравитация в открытом космосе7


В основном математикой. Ну и акселерометрами.
image.png324 Кб, 378x600
397 554755
анонасы, есть ли сайт/приложуха где можно просчитать траекторию движения объекта как созданного тобой, спутник допустим, так и естественного тела, поменяв вектор его движения, например луны или сатурна?

>>4735
гравитация либо есть, либо ты падаешь вместе с опорой, тобишь ее нет. разницы невесомости на уране и земляшке нет, а сила притяжения различна. P=gm
398 554757
>>4755

>есть ли сайт/приложуха где можно просчитать траекторию движения объекта как созданного тобой, спутник допустим, так и естественного тела, поменяв вектор его движения, например луны или сатурна?


Есть Universe Sandbox, там чисто посталкивать планеты, но не космические аппараты.

Чтобы всё вместе это GMAT - универсальный ответ на любой вопрос, не считая всяких STK и астродинамических фреймворков. Можешь в GMAT хоть Кербинскую систему засунуть. Правда 1) придется дохуя вбивать и 2) чтобы именно добавить планете пинок в м/с - придется питоном за API подёргать, по-моему NASA не предусмотрело подобного надругательства над природой в интерфейсе.
399 554761
>>4755
Я, видимо, неправильно сформулировал вопрос
Интересует именно открытый космос. Как в открытом космосе определить степень притяжения к звезде, черной дыре7
6ae85e3bc3ef2214d1e60485926c65c4.jpg798 Кб, 2100x2010
400 554762
Хочу узнать мнение экспертов по тому, какой способ разгона до околосвета (хотя бы 5%) самый дешёвый, с учётом современных технологий, если учитывать как и стоимость самой ёбы, так и цену за условный разгон 10 тонн на 1 миллипроцент от световой.
401 554765
>>4761
Никак.
Свободное падение, орбита и отсутствие притяжения ничем не отличаются.
402 554767
>>4765
Нипонил.. Но ведь с расстоянием сила тяготения уменьшается?.. И это никак не фиксируется?
403 554768
>>4762
Взрыволет на едренбатонах
404 554769
а
405 554770
406 554772
>>4768
Ну по-хорошему это же пиздец дорого. Каждый раз взрыволёт высирает за собой ядерную бомбу, которая не очень дешёвая, а таких бомб нужны сотни. Даже если мы уберём ебанутые километровые медные полусферы, как в Орионе, все системы охлаждения щита + сам щит + другая радиационная защита будут скорее всего охуенно дорогими.
407 554773
>>3800
ПХА сам по себе может взрываться (и гореть)
408 554778
>>4761
>>4735
Напрямую. Т.е. по ускорению, которое придает эта гравитация другим объектам с заранее известной массой. Принцип эквивалентности гласит, что гравитацию ничем не отличить от ускорения. То есть просто смотришь, как объект с заранее известной массой и скоростью отклоняется от прямой, и выводишь суммарное ускорение свободного падения (а следовательно и гравитация) в данной точке пространства.

>В каких единицах измеряется7


Ускорение свободного падения измеряется в м/с² и является векторной величиной.

>>4765
Принцип эквивалентности это хорошо, но на практике на тебя будет действовать градиент гравитации, который никак его не нарушает и который можно измерить. Либо разницей в ускорении головы и ног, либо кручением аппарата от приливных сил.
409 554781
На какой высоте высотометр становится нинужен и собственно высота становится расстоянием? от чего зависит? Опять ж от гравитации?
410 554787
>>4772
ну так сделай многоразовую ядерную бомбу
411 554788
>>4772
А чего ты хотел? Если б всё было так просто, мы бы давно уже по альдебаранам летали. Забесплатно только гравиманёвры.
mayor164735955orig.jpg384 Кб, 1296x1944
412 554789
>>4787

>многоразовую ядерную бомбу


Употребляете?
413 554790
>>4787

>ну так сделай многоразовую ядерную бомбу


>>4789

>Употребляете?



Ну так то Дедал и летает на многоразовых аж термоядерных бомбах. Лазерная инерционная инициация реакции, вот это все.

Другое дело что хотя такую многоразовую термоядерную бомбу и придумали - сделать не получается и когда получится - неизвестно.
image.png5,2 Мб, 1296x1944
414 554791
>>4789
?етеялбертопу
415 554792
>>4790

>Дедал и летает на многоразовых аж термоядерных бомбах


ВТФ? Обычные термоядерки.
416 554799
>>4792

>Дедал и летает на многоразовых аж термоядерных бомбах


>ВТФ? Обычные термоядерки.


Щито? У дедала все механизмы инициации реакции многоразовые и являются частью корабля, в отличие от "одноразовых" атомных бомб. Расходуемая часть это куски замороженнного топлива.
image.png592 Кб, 638x338
417 554809
сап, анчоусы, хочу понять каких размеров должна быть картинка на орбите, чтобы ее было видно не вооруженным глазом и в бинокль?
418 554814
>>4781
Там, где начинается космос, т.е. на линии Кармана. 100 км.
419 554816
Поясните за пространство.
Вот, пространство-время искривляется, ладно. Но откуда берётся энергия для этого искривления? Или пространство можно изгибать типа "бесплатно", энергия не тратится?
И ещё вопросик, чем же отличаются две точки в пространстве?
Строя модель, мы всегда берём координатную сетку, но у Вселенной никакой сетки ведь нету?
420 554817
>>4781
все зависит от системы измерения, а еще высота это и есть расстояние
421 554818
>>4778
Градиент заметен только у компактных массивных объектв либо крупным длинным детектором.
Точечно гравитацию не померить.
422 554819
>>4809
Смотри угловое разрешение глаза и желаемое разрешение картинки, умножай и получай расстояние.
423 554820
424 554821
>>4818

>Градиент заметен только у компактных массивных объектв


Приливная стабилизация на орбите Земли это охуенно важный фактор в проектировании ИСЗ, например, и используется для стабилизации некоторых, и это при не особенно-то больших размерах порядка автомобиля. Один из вариантов станции Freedom вполне предлагался приливно-стабилизируемым, например.

> либо крупным длинным детектором. Точечно гравитацию не померить.


Естественно. Но при не-планковских размерах принципиально возможно. Всё зависит от разнесения и точности акселерометров.
425 554826
>>4821
Акселерометры не покажут ускорение, ты же в свободном падении. Тебе надо весы иметь и сравнивать силу между разными частями станции.
426 554854
>>4826
Правильно, акселрометры регистрируют вес
427 554859
>>4854
Каким это образом? Акселерометр точечный датчик, без реакции опоры никакого веса в свободном падении ты не обнаружишь.
428 554860
>>4507
Ты пидор
429 554864
>>4860
достойный аргумент для быдла
430 554896
Возможно ли сделать телескоп в виде простейшего конусного отражателя от фонарика? Понятно, что если смотреть глазами то изображение звезд будет в виде бублика, но ведь можно будет обработать изображение на компьютере и убрать искажения.
431 554898
>>4896
Зеркальный конус выйдет дороже и сложнее обычного параболического зеркала.
432 554899
>>4898
Его же даже не обязательно зеркалить, можно использовать эффект полного внутреннего отражения.
433 554900
>>4899
Чет мой личный дерьмометр подсказывает, что геометрия не сходится.
Попробуй углы посчитать.
434 554904
>>4896
а смысл? все равно это не увеличит количество принимаемого света так как оно считается по площади начала конуса, а переотражения только ухудшат всё
435 554927
>>4904

>оно считается по площади начала конуса


Эм, схуяли? Свет-то входит в широкую часть.
IPkjob-1.png9,4 Мб, 5608x3739
436 554930
Что тут за скопление в центре?
IPkjob-1.png9,4 Мб, 5608x3739
437 554931
IPkjob-2.png9,4 Мб, 5608x3739
438 554932
>>4930
дашчан шалит, не даёт пикчу запостить
sage 439 554938
ебучий дашчан не показывал мне пики, всё понятно
440 554944
Ученые гавной моченые думали, куда улетела тейя после столкновения с землей?
441 554945
>>4932
Вау. Ты на ксяоми фотографировал?
442 554946
>>4927

>Свет-то входит в широкую часть.


Ну так широкая часть и есть апертура, за лимит которой не прыгнешь. Проще сделать параболическое зеркало такого же диаметра. Оно будет меньше весить, его проще формовать, и искажений оно практически не даёт.

Я круче придумал. Берём радиотелескоп "кратерного" типа (Аресибо, Тьяньян), покрываем его изнутри маленькими зеркалами на поворотных рамках, как в солнечных электростанциях с нагревателем. Получаем супер-зеркало с адаптивной оптикой диаметром в сотни метров, способное в оптический диапазон, и относительно задёшево. Наведение осуществляется смещением приёмника и синхронизированным доворотом зеркал.
443 554955
>>4944
Она утонула.
444 554990
>>4946
А кратер нахуя? Ровная поверхность тоже подойдет.
445 555007
>>4990
Разве?
1YKWeuj.png458 Кб, 803x732
446 555020
Вопрос 1: Почему звезда Марса в небе выглядит красной, хотя сам Марс жёлтенький? Почему красное смещение осуществляется на участки в сраные миллионы километров, а не за десятки световых лет? В солнечной системе пустота межпланетная какая-то особенная, пыльная и тугая, что даже в оригинале синевато-жёлтый Юпитер с Земли видится бело-красно-орнжевым?
Вопрос 2: Какие наиболее разумные и веские аргументы есть в обозримом научном мире, оспаривающие теорию расширения пространства, но при это объясняющие имеющие оптические феномены?
447 555023
>>5020

>Вопрос 1


Потому что свет проходит через воздух.

>Вопрос 2


Никакие.
sage 448 555063
>>4945
Не моё.
449 555064
>>4896
Ты только что ренген-гамма телескопы.
450 555065
Вот была Тея и другая протопланета называемая по другому. Почему эту протопланету называют землёй?
451 555071
>>5065
Потому чт Воду забрал себе Нептун, Воздух - Юпитер, а Огонь - Солнце
452 555072
>>5065
Почему реки от истока до устья называют одним именем, хотя в них впадают другие притоки?
453 555085
>>5065

>Почему эту протопланету называют землёй?


Ее называют не из-за того что она протопланета или какие-то предшествующие глобальные события. Она всегла была грязью
Ты живешь на планете ГРЯЗЬ
Так повелось.
Предыдущие события не при делах, название прижилось до понимания астрономии
454 555087
>>5020

>сам Марс жёлтенький


Он красный.

>Вопрос 2


Tired light
455 555088
>>5087
Красным блещет.
Я завидую.
Я лично никак больше блещущей красноватым точкой его не видел.
А кто-то наблюдал целый диск.
Хотелось бы мне тоже.
456 555096
>>5087

>Tired light


ОБОСНУЙ @ ПОЯСНИ
inb4: в гугле забанен
457 555097
>>5087

>Tired light


No.
458 555104
>>5087
Усталость света это другое название феномена, но никак не объяснение.
459 555126
https://habr.com/ru/news/t/494516/

> На спутнике связи АМ6 проблемы с охлаждением



Чего? В космосе -273. Какие еще проблемы? Что именно я не понимаю?
460 555130
>>5126
У вакуума нет температуры, а температура тела определяется балансом поглощения, тепловыделения и излучения. Зато есть Солнце, которое шпарит в космосе даже сильнее, чем в полдень на экваторе. Тело нагревается Солнцем, при наличии внуренних источников тепла, вроде электронных компонентов, нагревается изнутри, и достигает той температуры, когда начинает излучать все собственное тепло. Ну то есть, скажем, получает 2000 ватт тепла от Солнца, 500 выделяется изнутри, в итоге температура будет такой, при которой тело будет излучать 2500 ватт. Интенсивность излучения растет в зависимости от четвертой степени температуры. При этом излучение работает отводит тепло намного медленнее, чем конвекция и теплопроводность, поэтому охлаждение космических аппаратов представляет собой проблему.
Но это для общего случая, тут же проблема в перераспределении тепла - компоненты спутника рассчитаны на то, что тепло будет от них отводиться. Грубая аналогия - хотя в комнате холодно и ПеКа не перегревается целиком, но от процессора отвалился кулер.
image.png136 Кб, 260x325
461 555136
>>5130

>от процессора отвалился кулер.


Тут дело в том, что ты его не прикрутил в мамку, а приклеил на термопасту.
Забавная аналогия.
Всё верно расписал, ты молодец.
462 555139
>>5126
В космосе напрямую обмениваться теплом не с кем. У спутника на орбите Земли бывает только излучательная теплопередача, на "том конце" которой находятся три объекта:
- Солнце (тысячи градусов, периодически закрывается Землей в зависимости от орбиты, занимает малый градус неба)
- Земля (примерно комнатной температуры в среднем, плюс-минус сапог)
- и фон (очень холодный, несколько кельвин, занимает большую часть неба)
Любая теплопередача идёт в направлении от теплого к холодному, и чем больше разница температур, тем она эффективней. На неё можно повлиять, покрывая спутник специальным покрытием, но Солнце все равно пересиливает всё остальное, прилично шпаря. Поэтому многие спутники имеют жидкостное охлаждение электронных нутрей, через трубки с теплоносителем сбрасывая тепло в радиатор, который отражает большую часть спектра Солнца, но в инфракрасном спектре является черным (и хорошо излучающим). Часто радиатором является тупо часть корпуса, который стерпит нагрев, в отличие от начинки. В более сложных случаях (например МКС) радиаторы представляют собой лепестки наподобие солнечных батарей.
image.png1,2 Мб, 1024x546
463 555140
>>5139

>и фон (очень холодный, несколько кельвин, занимает большую часть неба)


Всё небо, всё вокруг без исклчения. Все эти яркие/холодные пятна - сильно СИЛЬНО преувеличены, там флуктуации в долях Кельвина.
464 555153
>>5140

>Всё небо, всё вокруг без исклчения


Ну на низкой орбите приличную часть небосвода загораживает Земля. Вот на ГСО уже небольшую.
465 555154
>>5153
Ну это безусловно верно.
Я лишь говорил про то, что нет у небосвода какого-то места где нуль кельвинов и можно туда охлаждаться. Вся вселенная это минимум 4К без исключения (кроме наших криолабораторий)
466 555155
>>5154
Ясен хуй, я про то и не говорил. А охлаждаться таки есть куда, 4 кельвина это офигенно холодно, если у тебя не йоба-телескоп с матрицей при почти абсолютном нуле, а обычная электроника. Суммарно Солнце в наших краях солнечной системы забарывает фон (с точки зрения термодинамического баланса), но у Солнца полная энергия рассеяна по спектру, а излучает нагретый спутник только в дальнем инфракрасном. За счет этой разницы можно эффективно сбрасывать лишнее тепло. (и даже удерживать водород от выкипания при 20K безо всяких зонтиков, см. EUS)
467 555156
>>5155
Добра тебе.
15837698922610.png114 Кб, 680x559
468 555177
Я слышал что локхид в 2020 предоставит прототип двигателя будущего, может ли быть так, что нефтяной пиздец начался из-за этого?
469 555178
>>5177
Нет, лягух, вообще никакой связи нет и не может быть.
470 555180
>>5178
А что на счёт двигателя локхида?
471 555181
>>5180
Просто очередная йоба. Локхид их постоянно делает.
472 555183
>>5181
Я просто читал на вики что локхид обещал в 2020 предоставить образец йоба движка на йоба технологии, который можно катать в грузовике, обеспечит город энергией и экологичный.
473 555184
>>5183
А, тот самый, что каждый год последние пять лет обещают что ли? Не движок, а реактор?
Ну продолжай следить, в следующем-то точно сделают.
474 555185
>>5184
Ну лан, я просто думал...
475 555186
>>5185
Бывает. Не стесняйся задавать вопросы. От этого ты хуже не станешь.
476 555187
>>5186
Мы случайно не знакомы?
httpss3.amazonaws.comthe-drive-stagingmessage-editor2F15242[...].jpg76 Кб, 800x601
477 555197
>>5183
Этот реактор похоже все. Случилась классика - очередная "мы тут придумали и сделаем быстро, просто и недорого реактор" уже превратилась в "через пару поколений лабораторных установок мы начнем приближаться к термоядерным параметрам", а реактор вырос до пары десятков метров и пары тысяч тонн.
Планировали в 2017 прототип, в 2022 - коммерческий продукт.
Пикрелейтед их роадмап, T5 начали делать в прошлом году. Потом будут Т6, Т7, и только потом прототип. Теперь это типичный реактор-будет-через-20-лет.
478 555213
>>5197
А есть ли список таких дирижаблей будущего по разным отраслям?
479 555214
>>5197
Вообще-то в прошлом году Локхид объявил постройку новых зданий в сканк воркс ради этой хуйни
480 555215
>>5214

>сканк воркс


Это что-то непристойное?
image.png815 Кб, 800x600
481 555216
>>5215
Отдел тайных и передовых разработок у Локхида
482 555217
>>5214
И что? Я же не писал, что проект прекращен. Мало того, я же даже написал, что в прошлом году начали работу над следующей лабораторной установкой - Т5. Но от Т5 до реактора как до Луны, они в принципе подойдут к термоядерным параметрам даже не на следующем а через поколение.
Блин, по их планам от 2014 года, у них уже сегодня должен быть работающий прототип реактора, а они пока только с низкотемпературоной плазмой работать учаться.
Да, весьма вероятно, что они будут дальше работать. Но нужно понимать, что теперь это типичный термоядерный проект, который будет разрабатываться десятилетия с неясными перспективами. Весь апломб, что "мы коммерческая фирма, мы те самые Сканк Воркс, которые умеют решать задачи, и сейчас мы покажем этим государственным грантососам, как надо работать" накрылся пиздой. Нет, ну можно в них верить, но пока они повторяют путь всех предыдущих разработчиков термояда, которые начав с отличной идеи и простого прототипа намертво увязают в болоте, с рекатором-через-двадцать-лет.
483 555218
>>5217
вы находитесь вот здесь ...
484 555219
>>5218

>сканк воркс в 21 веке


>этот мем


Последней 80/20 годнотой SW была стелс-табуретка, скунс сдулся когда ушёл Рич. Не стоит применять это к ним.
485 555220
>>5213
На этом не один многотомник можно написать.
Про космонавтику есть например прекрасный бложик https://falsesteps.wordpress.com/ , автор даже в книгу его оформил
486 555221
>>5219
посмотрим че ты скажешь когда ср-72 выкатят
.jpg99 Кб, 480x321
487 555222
>>5215
https://www.amazon.com/dp/0316743003
Читни книжца от бати, много инженерно-управленческих баек узнаешь, заодно и что такое сканк воркс поймешь.
488 555223
>>5222
https://www.amazon.com/dp/B006Q2D1JI/
Ну и от дида тоже, канеш
489 555235
>>5177
Да, это так.
мимо форсер варпдрайва от локхидов
490 555236
>>5235
иди нахуй
491 555245
>>5187
Я крайне сомневаюсь в такой возможности.
492 555251
>>5155

>нагретый спутник


Но ведь мы его охлаждаем, поэтому не нагретый? Нет ли тут обратных связей в смысле охлаждаем спутник->меньше излучает->больше нагревается->больше излучает?

Точнее не так, такие ОС, конечно, есть, я про смещение точки излучения - с падением температуры спутника он начинает излучать ближе к видимому свету, менее эффективно, так? А при нагреве - наоборот?

Блядь, еще стакан виски и я термодинамическое равновесие!
493 555254
>>5251

>Нет ли тут обратных связей в смысле охлаждаем спутник->меньше излучает->больше нагревается->больше излучает?


В какой-то точке наступит баланс, вот и все. Если у радиатора отводящая поверхность большая, он тебя заморозит, если маленькая - на солнце нагреешься.

>с падением температуры спутника он начинает излучать ближе к видимому свету, менее эффективно, так? А при нагреве - наоборот?


Не так. Горячее - ближе к видимому. Чтобы хотя бы начать излучать в видимом свете, надо докрасна раскалиться, а чтобы конкретно заизлучать - как само Солнце.
494 555255
>>5217
Самая жопа в заявлении - термоядерный реактор в контейнере? Это принципиально невозможно БЕЗ ЕБАНОГО ОБЛУЧЕНИЯ ВСЕГО ВОКРУГ.
В реальности не может быть высокоэнергетических реакций без высокоэнергетических частиц которые хуярят кругом.
Если они пытаются выдумать новый вид защиты от радиации - пусть пытаются, достойное начинание, хотя без бэка элементарной физики.
495 555256
>>5255
Не соглашусь. Синтез возможен и без нейтронов в продуктах реакции, об том и токамаки с ITERами.
496 555257
>>5256
Подробнее про синтез без нейтронов.
Ты же в курсе что там не 1+1 = 2, а дохуища реакций, друг?
497 555258
>>5256

>об том и токамаки с ITERами


Да ты сё
498 555259
>>5256
Невозможен. Даже анейтронным реакциям всегда сопутствуют нейтронные и дают приличный поток, пусть и в него уходит небольшая доля энергии реакции, но он есть. Токамаки с итерами - про синтез вообще, а не безнейтронный синтез. Анейтронные реакции поджигаются гораздо сложней, тут бы хоть с обычными выход обеспечить, а ты сразу про гелий-3.

>>5255
Поток-то направленный, можно сфокусировать в мишень. Будет хуярить в регулярно меняемую тряпочку.
1343566211895.jpg54 Кб, 363x500
499 555260
>>5259
Взбурлил.
500 555264
>>5259

>Даже анейтронным реакциям всегда сопутствуют нейтронные


11B + H

>Поток-то направленный


Щито-щито?
501 555265
>>5259
Какой направленный поток, о чем ты вообще?

При слиянии дейтерия и трития получается нестабильное ядро гелия-5, которое моментально испускает нейтрон в случайном направлении и отдачей отталкивается в противоположном. Никакой направленности там нет, плазма хуярит нейтронами во все стороны.
502 555268
>>5259

>Поток-то направленный


Это обоснуй.
Как ты направляешь выход ядерной реакции?
503 555281
>>5218
Эх, я обожаю эту фразу. Еще можно написать про престьянина 19 века, не понимающего зачем нужен автомобиль, когда есть лошадь. И про тех, кто ищет отговорки, а кто-то ищет решение. И написать, главное, так можно про что угодно, про реактор, про ЕМ-драйв, про вечный двигатель.
У Маска эта надпись использовалась между, внезапно, результатами. А у SW - между невыполненными обещаниями.
То есть пока выглядит так - в 2011 прыгнул Grasshopper сейчас 2017 год, уже должен был слетать Falcon Heavy, через 5 лет должен лететь StarshipBFR, а список все еще выглядит так:

Вот прыгнет Grasshopper тогда и поговорим
=вы находитесь здесь=
Вот когда полетит их Falcon 1, тогда поговорим.
Вот когда будут клиенты, тогда поговорим.
Вот когда будет контракт с NASA, тогда поговорим.
Вот когда полетит их Falcon 9, тогда поговорим.
Вот когда построят свой корабль, тогда поговорим.
Вот когда долетит он до МКС, тогда поговорим.
Вот когда будут запускать геостационарные спутники, тогда поговорим.
Вот когда придумают, как сажать ракеты, тогда поговорим.
Вот когда посадят хоть одну, тогда поговорим.
Вот когда посадят на баржу, тогда поговорим.
Вот когда начнут сажать после вывода на ГПО, тогда поговорим.
Вот когда полетит б/у ракета, тогда поговорим.
Вот когда запустят свой Falcon Heavy, тогда поговорим.
Вот когда вернут обтекатель, тогда поговорим.
Вот когда сделают свой Crew Dragon, тогда поговорим.
Вот когда повторно используют обтекатель, тогда поговорим.
Вот когда начнут возить людей, тогда поговорим.
Вот когда научатся сажать корабль без парашютов, тогда поговорим.
Вот когда запустят корабль на Марс, тогда поговорим.
Вот когда он сядет на Марсе, тогда поговорим.
Вот когда сделают марсианский транспорт, тогда поговорим.
Вот когда высадят людей на Марсе, тогда поговорим.
Вот когда колонизируют Марс, тогда поговорим.
Вот когда терраформируют Марс, тогда поговорим.
503 555281
>>5218
Эх, я обожаю эту фразу. Еще можно написать про престьянина 19 века, не понимающего зачем нужен автомобиль, когда есть лошадь. И про тех, кто ищет отговорки, а кто-то ищет решение. И написать, главное, так можно про что угодно, про реактор, про ЕМ-драйв, про вечный двигатель.
У Маска эта надпись использовалась между, внезапно, результатами. А у SW - между невыполненными обещаниями.
То есть пока выглядит так - в 2011 прыгнул Grasshopper сейчас 2017 год, уже должен был слетать Falcon Heavy, через 5 лет должен лететь StarshipBFR, а список все еще выглядит так:

Вот прыгнет Grasshopper тогда и поговорим
=вы находитесь здесь=
Вот когда полетит их Falcon 1, тогда поговорим.
Вот когда будут клиенты, тогда поговорим.
Вот когда будет контракт с NASA, тогда поговорим.
Вот когда полетит их Falcon 9, тогда поговорим.
Вот когда построят свой корабль, тогда поговорим.
Вот когда долетит он до МКС, тогда поговорим.
Вот когда будут запускать геостационарные спутники, тогда поговорим.
Вот когда придумают, как сажать ракеты, тогда поговорим.
Вот когда посадят хоть одну, тогда поговорим.
Вот когда посадят на баржу, тогда поговорим.
Вот когда начнут сажать после вывода на ГПО, тогда поговорим.
Вот когда полетит б/у ракета, тогда поговорим.
Вот когда запустят свой Falcon Heavy, тогда поговорим.
Вот когда вернут обтекатель, тогда поговорим.
Вот когда сделают свой Crew Dragon, тогда поговорим.
Вот когда повторно используют обтекатель, тогда поговорим.
Вот когда начнут возить людей, тогда поговорим.
Вот когда научатся сажать корабль без парашютов, тогда поговорим.
Вот когда запустят корабль на Марс, тогда поговорим.
Вот когда он сядет на Марсе, тогда поговорим.
Вот когда сделают марсианский транспорт, тогда поговорим.
Вот когда высадят людей на Марсе, тогда поговорим.
Вот когда колонизируют Марс, тогда поговорим.
Вот когда терраформируют Марс, тогда поговорим.
504 555282
возможно ли выйти в открытый космос в трусах? чтобы из тебя не вытянуло кислород нужно просто заткнуть уши, жеппу, может быть на глаза надеть очки как у дайверов хз нос и зажать рот и, по идее, это ничем не отличиться от обычной задержки дыхания. радиация, излучения лубая другая хуйня ничего тебе за эти 3 минуты не сделает. я прав?
505 555285
>>5281
Сектант не палится
Space Suit Testing.webm2,4 Мб, webm,
426x240, 3:27
506 555286
>>5282
Зажать нос и рот ты не сможешь вообще, слишком сложно в вакууме. Через ~15 секунд ты потеряешь сознание, получишь синяки и гематомы по всему телу, начнется декомпрессионная болезнь. После ~40 остановка сердца и всякие необратимые последствия. В целом, считается что больше 90 секунд после резкой разгерметизации не выжить. В 1966 году у NASA техник в вакуумной камере разгерметизировался при испытаниях скафандра, еле успели вытащить. Это классический случай.
507 555288
>>5286
Чёт youtube-dl вытащил без звука.
https://www.youtube.com/watch?v=KO8L9tKR4CY короче
508 555292
>>5286
ты описал то, что будет с человеком без задержки дыхания, но этого не произойдет в моем случае. тут кислород будет в легких и кровь будет забираиь его пополняя его, а значит кислород к мозгу поступать будет и значит сознание не потеряется.
509 555293
>>5292
Ты не удержившь воздух в легких, у тебя гортань не герметичный клапан. Он способен держать воздух при разнице давлении вверх, когда снаружи больше давления, но вниз не держит, и сразу весь воздух из легких стравит.
Предлагаю построить вакуумную камеру и потестить наши гипотезы на китайцах.
510 555295
>>5293
бля это вообще не проблема для этого хоть мини скафандр можно построить в виде маски даже китайцы не нужны
511 555297
>>5295
Не мешай моим планам геноцида китайцев, сволота!
512 555302
>>5297
гэй
513 555305
>>5292

>ты описал то, что будет с человеком без задержки дыхания


Не, я описал что будет при резкой декомпрессии до вакуума. Задерживай или не задерживай, эта хуйня тебя убьет гораздо быстрее чем отсутствие кислорода
514 555311
>>5285
Что, у нас теперь есть секта "неверующих во всесилие Сканк Воркс?"
515 555318
>>5311
ХЗ. Но верить в маняобещания не надо. Неудивительно, что за веру в неподтвержденные маняобещания записывают в сектанты.

другой анон
516 555323
>>5318

>Неудивительно, что за веру в неподтвержденные маняобещания записывают в сектанты.


Ващет наоборот, тут сейчас меня пытаются записать в секту за отсутствие веры в маняобещания.
517 555324
>>5305
что убьет то? перепад в 1 атмосферу?
image.png132 Кб, 300x164
519 555333
520 555340
Вопрос по астероиду Didymos B

Вы тут в спейсаче уже знаете его реальную траекторию или это пока не известно?

Может ли так быть, что NASA хочет нанести по нему удар не в качестве тренировки, а потому что эти великолепные 160м в диаметре собираются ебнуть по нашей уютной планете?
521 555341
>>2819 (OP)

>Пик4


Почему "греков" и "троянцев" много, а "хильдов" меньше? (Или по массе эти группы одинаковые?) Логика подсказывает, что хильдов должно быть столько же сколько гомеровских астероидов.
522 555342
>>2819 (OP)

>Пик1


Если я еще не заебал.
А нет ли примерно такого же, но в виде гифки или вебки. Вот эти вот все ваши гравитационные ямы в движении очень завораживают
523 555346
>>5340
С учетом того, что это спутник астероида диаметром порядка 800 метров, а расстояние между ними около 1,2 километра, то нет. Потому как если ебнут, то ебнут оба, и 160 метров Дидимуна будут представлять из себя меньшую из проблем.
А так, в ближайшие 100 лет не будет приближаться ближе 7 миллионов километров.
15852617747960.jpg95 Кб, 640x640
524 555372
>>5346

>800 метров,


0_0 ух бля

>спутник


Аааа, теперь понятно, почему именно этот. Можно будет точно высчитать эффект воздействия насовской хуйни, да?

Круто, спасибо
525 555374
>>5346
Где же организации по защите диких астероидов?
526 555375
>>5324
Да. Это оче дохуя, в частности для твоей кровеносной, лимфатической и прочих жидкостных систем.
527 555379
528 555420
>>5305
Пиздец ты реально тупой.
Поройся на ютубе. Я может ссылку принесу потом, там один перец на пальцах обьясняет, от чего ты сдохнешь в космосе и это не от перепада давления и кровькишкираспидорасило.
В барокамерах с космонавтами уже случалось подобное и нихуя никого не разорвало.
529 555421
>>5375
На глубине 10м, давление 2атм, всплывая быстро на поверхность, даже кессонку не получишь. Перепад 1 атм.
Почему вы блять не можете в гугл суки? Это реально тред тупых, не вопросов, а просто тупых.
530 555422
>>5282

>возможно ли выйти в открытый космос в трусах?


Будешь замерзать из-за быстрого испарения влаги с поверхности кожи.

>нос и зажать рот и, по идее, это ничем не отличиться от обычной задержки дыхания


Именно так.

>радиация, излучения лубая другая хуйня ничего тебе за эти 3 минуты не сделает


За 3 минуты не успеют. Умрёшь ты не от радиации, не от перепада давления, твою тушку не распидорасит как в кино со Шварцнеггером.

Самое интересное начнётся, когда ты уже не сможешь задерживать дыхание и сделаешь вдох или выдох, то есть твой организм попытается дышать. Сразу же из лёгких улетучится весь объём газа, они станут абсолютно пустыми.
Так как давление в твоей кровеносной системе выше чем снаружи, то все газы содержащиеся в крови, кислород, азот, углекислый газ, начнут проникать из крови через альвеолы в лёгкие. У тебя изо рта повалит пар, разумеется потому что в выдыхаемых газах будет содержаться вода в том числе. Получится обратный процесс, не насыщение крови газами, а дегазация крови. Примерно через 10 секунд ты потеряешь сознание, примерно через 30 секунд, сердце прогонит всю кровь по кровеносной системе и в ней не останется ни какого кислорода, азота, углекислого газа и ты умрёшь. Через три часа твоя тушка остынет настолько что полностью промёрзнет, вся не испарившаяся вода, превратится в лёд.
15837037452480.jpg79 Кб, 800x450
531 555427
>>5420
Я тебе дал именно то видео про барокамеру с космонавтами, о котором ты говоришь, блять! И цитирую выводы насы из репорта об инциденте.

>>5421
Почему ты не можешь в мозг? При всплытии ты доходишь до нормального давления. Да блять, это тред тупых по ходу. Декомпрессия до вакуума принципиально отличается от декомпрессии до атмосферного давления.

>>5422

>когда ты уже не сможешь задерживать дыхание


Ты его не сможешь сдерживать вообще, уже при сколько-нибудь заметном падении, не то что в вакууме.
532 555438
Правильно ли я понимаю что из-за кризиса/короновируса америка нихуево так просядет финансово и полет на луну придется откладывать на 2030-2040 годы?
533 555475
>>5422

> Именно так.


Разве не разорвет легкие к хуям от попытки задержать дыхание ?
atlante.jpg50 Кб, 500x674
534 555521
Перекат обратно в античность:

>>555520 (OP)
>>555520 (OP)
>>555520 (OP)
Тред утонул или удален.
Это копия, сохраненная 6 августа 2020 года.

Скачать тред: только с превью, с превью и прикрепленными файлами.
Второй вариант может долго скачиваться. Файлы будут только в живых или недавно утонувших тредах. Подробнее

Если вам полезен архив М.Двача, пожертвуйте на оплату сервера.
« /spc/В начало тредаВеб-версияНастройки
/a//b//mu//s//vg/Все доски